Sei sulla pagina 1di 87

Geometra Moderna I

Notas de clase
Versi on preliminar
Rodolfo San Agustn Chi
Departamento de matem aticas
Facultad de Ciencias
Universidad Nacional Autonoma de Mexico
rodolfomeister@ciencias.unam.mx
24 de enero de 2013
ii Rodolfo San Agustn Chi
Prefacio
Yo nunca permit que la escuela interriera con mi educacion.
Mark Twain
1
El hombre nace ignorante, no est upido. Se hace est upido por la educaci on.
Bertrand Russell
2
Muchos cursos y textos tanto de geometra como de otras ramas de la matem atica
acogen con particular interes la resolucion de problemas y este, de alguna forma, no es la
excepcion. Nuestra idea es la de enfatizar el car acter autoformativo que tiene este tipo de
estudios. Es decir, en primer lugar, seguimos un formato como el de los muchos problemarios
que ofrece la literatura (v.gr [AC07], [BMGO02a] y [BMGO02b] en la bibliografa al nal
de estas notas): para cada captulo damos solamente las deniciones m as pertinentes, los
teoremas estructurales b asicos y algunas construcciones, ejemplos o casos ilustrativos. Sin
embargo, en tanto que el material (tambien) est a dirigido a quien se inicia en el estudio
de una carrera cientca, hemos considerado una presentacion un tanto m as formal del
asunto. As, tanto el material expositorio como el que se deja a las secciones de ejercicios
se presentan como problemas, teoremas, construcciones, etc. requiriendo, por supuesto, de
sus respectivas demostraciones o justicaciones.
El curso se llama Geometra Moderna pues se enfoca al estudio de la geometra cuyo
desarrollo sigui o al renacimiento europeo (mediados del siglo XV) y que llega hasta nales
del siglo XVIII, o incluso, hasta principios del siglo XIX
3
, incluyendo, en particular, las
epocas de la ilustracion y la napoleonica
4
.
1
Samuel Langhorne Clemens, mejor conocido por su seudonimo literario: Mark Twain (1835 1910).
2
Bertrand Arthur William Russell (1872 1970).
3
Esto es, grosso modo, la llamada epoca moderna en la historiografa europea
4
El progreso y el perfeccionamiento de las matematicas est an ntimamente ligados a la
prosperidad del Estado.
Napoleon Bonaparte
As, citando diversas fuentes:
...Cuando [Napoleon] llego a emperador, apoy o vigorosamente la ciencia francesa tratando que compitiera
exitosamente con la inglesa.
...Fue el quien impuls o la educaci on superior y someti o a las escuelas a un control centralizado ampliando
el sistema educativo libre, de manera que cualquier ciudadano pudiera acceder a la ense nanza secundaria sin
que se tuviera en cuenta su clase social o religion.
i
ii Rodolfo San Agustn Chi
Desde la epoca de Euclides, y quizas ya anteriormente, el llamado metodo o principio
de superposici on ha sido una herramienta did actica muy fructfera, directa e intuitiva para
iniciarse en el estudio de la geometra. Sin embargo, por el nivel que desean alojar estas
notas, nos ha parecido conveniente manejarnos con una mayor formalidad en las discusiones
al respecto
5
. As, nuestro enfoque hace referencia explcita a los sistemas de Euclides, por
una parte, y al de Hilbert, por la otra.
Cabe aclarar que no se trata de hacer un tratamiento completamente basado en los
fundamentos de la geometra, sino solamente de aclarar, hasta un cierto punto, los bases
que empleamos en nuestro estudio de la misma. Por ejemplo, con los grupos de axiomas de
incidencia y de congruencia del sistema de Hilbert podremos analizar con cierto cuidado
como trabaja el ya mencionado principio de superposici on.
Por otra parte, el material de la geometra sintetica en s, despues de varios milenios de
desarrollo, es muy abundante
6
. As, hemos preferido hacer menci on unicamente a algunas
de nuestras fuentes m as directas. Estas, a su vez, llevaran al lector a sus propias fuentes
y as, sucesivamente. Ademas, huelga escribir, tambien en este aspecto la web esta con
nosotros.
No parece necesario remarcar, una vez mas, el como las aplicaciones, tanto dentro como
fuera de la matem atica
7
, han puesto de relieve los aspectos combinatorio y computacional
de la geometra. Tampoco queremos hacer, ni siquiera, una introducci on a las ahora llamadas
geometras nitas o combinatorias. Solamente destacaremos el punto en algunos momentos
cuya relevancia, belleza o, incluso, la suerte, han llamado nuestra atenci on.
El captulo primero, inicia con un peque no estudio, aunque esencial y suciente para
nuestras necesidades, de la congruencia de tri angulos. En segundo lugar consideramos angu-
los inscritos en una circunferencia y damos una primera caracterizaci on de los cuadrilateros
cclicos En la tercera parte de este captulo entramos a la geometra moderna mediante
la aritmetica de segmentos orientados en una recta. Finalmente, incluimos un apendice a
este captulo, como referencia, con las listas completas de los postulados de Euclides y los
axiomas de Hilbert.
Puede decirse que el captulo segundo es el coraz on de estas notas. En el se estudia la
semejanza de tri angulos la cual gua no s olo a dicho captulo, sino a todos los subsiguientes.
El captulo tercero nos da algunos criterios de colinealidad (de puntos) y concurrencia
(de rectas). Este material se puede enmarcar ...[...]
...Cualquiera que haya sido la capacidad geometrica de Napoleon, es merito suyo haber revolucionado de
tal forma la ense nanza de las matematicas en Francia, que seg un varios historiadores,
...sus reformas fueron las causantes de la oracion de matematicos orgullo de la Francia decimon onica.
5
Por otra parte, recientemente se ha argumentado, por ejemplo, en el libro de Cromwell ([Cro97]) la
reserva que aparentemente tenia el mismo Euclides para emplear este metodo.
6
Como una referencia, en ([Eve69], p. 63) se menciona que En 1906 Max[imiliano] Simon (en su

Uber
die Entwickelung der Elementar-geometrie im XIX Jahrhundert) intento la formacion de un cat alogo de
aportaciones a la geometra elemental hechas durante el siglo XIX; se ha estimado que este cat alogo contiene
mas de 10,000 referencias!...
7
Baste mencionar dentro de estas disciplinas a la teora de codigos y la criptografa, as como los dise nos
combinatorios, la rob otica, la vision computarizada, la teora de grupos y el algebra computacional.
Notacion general
Un lenguaje no es un repertorio arbitrario de smbolos,
sino una manera de sentir la realidad.
Jorge Luis Borges
Denotaremos, tal como es muy frecuente en el caso de la geometra plana, a los puntos
con letras may usculas: A, B, C, etc., a las rectas con letras min usculas: a, b, c, etc. y a los
angulos con letras griegas , , , etc.
El smbolo denota
congruencia, tanto de segmentos y de angulos, como de tri angulos.
(ABC) area del tri angulo con vertices A, B y C.
semejanza de tri angulos.
. . . 2 n de la demostracion.
AB | CD las rectas AB y CD son paralelas.
( )
el tri angulo denido por las condiciones ( ).
Analogamente ( ), circ( ), etc.
El smbolo se utiliza consistentemente en fsica para referirse a la proporcionalidad
de dos cantidades; por ejemplo, presion y temperatura.
Recordemos que, siguiendo a Hilbert
8
, se puede manejar al segmento AB con la relaci on
indenida estar entre y los axiomas de orden (ver el apendice al nal del captulo I) o bien,
conjuntistamente, como sigue:
AB :=

AB

BA.
8
David Hilbert(1862, Konigsberg, Prusia Oriental 1943, Gottingen, Alemania) fue un matematico alem an,
reconocido como uno de los mas inuyentes del siglo XIX y principios del XX. Establecio su reputacion
como gran matematico y cientco inventando o desarrollando un gran abanico de ideas, como la teora de
invariantes, la axiomatizacion de la geometra y la noci on de espacio de Hilbert, uno de los fundamentos
del an alisis funcional. Hilbert y sus estudiantes proporcionaron partes signicativas de la infraestructura
matematica necesaria para la mec anica cu antica y la relatividad general. Fue uno de los fundadores de la
teora de la demostraci on, la l ogica matematica y la distincion entre matematica y metamatem atica. En el
congreso internacional de matematicas de 1900 presento un conjunto de problemas que establecieron el curso
de gran parte de la investigaci on matematica del siglo XX.
iii
iv Rodolfo San Agustn Chi
sin embargo, a lo largo de estas notas pr acticamente no ser a necesario recurrir a esta no-
tacion.
Con respecto al area de un tri angulo, solamente emplearemos el hecho de que dicha
area se puede calcular con la formula que sigue:
(tri angulo) =
1
2
base altura (1)
Para muchos matem aticos, 2 es el smbolo de Halmos
9
. Algunos otros se lo atribuyen
a Conway
10
.
9
Paul R. Halmos (1916-2006) A este matematico tambien se le atribuye la abreviatura de la expreci on
en ingles f and only if.
10
de la Wikipedia: John Horton Conway (nacido en Liverpool, Gran Breta na el 26 de diciembre de 1937).
Prolco matematico activo en la teora de conjuntos (teora de conjuntos nitos), teora de nudos, teora
de n umeros, teora de juegos y teora de codigos. Se formo en la Universidad de Cambridge. Entre los
matematicos acionados, quizas es mas conocido por su teora de juegos combinatorios, en particular por
ser el creador en 1970 del juego de la vida. Tambien es uno de los inventores del juego del drago, as como
del Phutball y ha realizado an alisis detallados de muchos otros juegos y problemas, como el cubo Soma.
Invento un nuevo sistema numerico, los n umeros surreales, los cuales se encuentran estrechamente relaciona-
dos a ciertos juegos y han sido objeto de una novela matematica por Donald Knuth. Tambien ideo una
nomenclatura para n umeros excesivamente largos, la nowiki o echa encadenada de Conway. Actualmente es
profesor de matematicas en la Universidad de Princeton. En 1981 fue elegido miembro de la Royal Society.
Ha escrito varios libros de divulgacion incluyendo Sobre n umeros y juegos (ISBN 0-12-186350-6) y Maneras
de ganar sus juegos matematicos. Visito Mexico en julio de 2010.

Indice general
1. Introducci on 1
1.1. Congruencia de tri angulos . . . . . . . . . . . . . . . . . . . . . . . . . . . . 3
1.2. El teorema de Pit agoras . . . . . . . . . . . . . . . . . . . . . . . . . . . . . 9
1.3. Cuadrilateros cclicos . . . . . . . . . . . . . . . . . . . . . . . . . . . . . . . 9
1.3.1.

Angulos inscritos en una circunferencia . . . . . . . . . . . . . . . . 9
1.3.2. Cuadrilateros cclicos . . . . . . . . . . . . . . . . . . . . . . . . . . . 11
1.4. Segmentos de recta dirigidos . . . . . . . . . . . . . . . . . . . . . . . . . . . 13
1.4.1. Raz on de partici on de un segmento dirigido . . . . . . . . . . . . . . 15
1.5. Apendice . . . . . . . . . . . . . . . . . . . . . . . . . . . . . . . . . . . . . 19
1.5.1. Principios de Euclides . . . . . . . . . . . . . . . . . . . . . . . . . . 19
1.5.2. Axiomas de Hilbert . . . . . . . . . . . . . . . . . . . . . . . . . . . 20
2. Semejanza 23
2.1. Semejanza de tri angulos. . . . . . . . . . . . . . . . . . . . . . . . . . . . . . 23
2.2. Teorema de Tolomeo . . . . . . . . . . . . . . . . . . . . . . . . . . . . . . . 27
2.3. Homotecia . . . . . . . . . . . . . . . . . . . . . . . . . . . . . . . . . . . . . 29
2.3.1. Algunas propiedades b asicas de la homotecia: . . . . . . . . . . . . . 30
2.3.2. Circunferencias homoteticas . . . . . . . . . . . . . . . . . . . . . . . 31
2.3.3. Puntos hom ologos y antihom ologos . . . . . . . . . . . . . . . . . . . 33
2.3.4. Circunferencia de similitud . . . . . . . . . . . . . . . . . . . . . . . 34
2.3.5. Crculo de Apolonio . . . . . . . . . . . . . . . . . . . . . . . . . . . 37
3. Algunos teoremas de concurrencia y de colinealidad 39
3.1. Teoremas de Ceva y Menelao . . . . . . . . . . . . . . . . . . . . . . . . . . 40
3.2. Tri angulos en perspectiva . . . . . . . . . . . . . . . . . . . . . . . . . . . . 47
4. Relaciones arm onicas 51
4.1. Division arm onica. . . . . . . . . . . . . . . . . . . . . . . . . . . . . . . . . 51
4.2. Transversal de un haz arm onico . . . . . . . . . . . . . . . . . . . . . . . . . 54
4.3. Hileras arm onicas en perspectiva . . . . . . . . . . . . . . . . . . . . . . . . 55
4.4. Rectas conjugadas perpendiculares . . . . . . . . . . . . . . . . . . . . . . . 56
4.5. Curvas ortogonales . . . . . . . . . . . . . . . . . . . . . . . . . . . . . . . . 57
4.6. Cuadrangulos y cuadrilateros completos . . . . . . . . . . . . . . . . . . . . 59
4.6.1. . . . . . . . . . . . . . . . . . . . . . . . . . . . . . . . . . . . . . . 60
v
vi Rodolfo San Agustn Chi
5. La circunferencia de los nueve puntos y la recta de Simson 65
5.1. Grupos ortocentricos . . . . . . . . . . . . . . . . . . . . . . . . . . . . . . . 65
5.2. La circunferencia de los nueve puntos . . . . . . . . . . . . . . . . . . . . . . 69
5.3. Recta de Euler . . . . . . . . . . . . . . . . . . . . . . . . . . . . . . . . . . 70
5.4. La recta de Simson . . . . . . . . . . . . . . . . . . . . . . . . . . . . . . . . 72
Captulo 1
Introduccion
En este captulo reunimos algunos resultados de geometra elemental que nos ser an
de utilidad. Iniciamos con un peque no estudio, aunque esencial y suciente para nuestras
necesidades, de los teoremas principales de congruencia de tri angulos. En segundo lugar,
revisamos algunas propiedades de los angulos inscritos en una circunferencia, denimos
los cuadrilateros cclicos y, tras algunas consideraciones de tipo combinatorio sobre dichos
objetos, llegamos a una primera caracterizaci on de estos ultimos.
A partir de la pen ultima parte de este captulo hacemos contacto con la geometra mo-
derna mediante la nocion de segmento dirigido y daremos algunas identidades fundamentales
en la aritmetica de segmentos en una recta.
En geometra, como en tantas otras cosas de la vida, puede ser muy util el suponer o
postular como verdadero algo que parece serlo. Si, adem as, esto que postulamos as es o
parece ser de lo m as simple, tenemos un ambiente de trabajo que nos permite referirnos
claramente a estos principios
1
. Esta parece haber sido la idea de Euclides
2
al redactar sus
Elementos
3

4
Considero un conjunto de 5 postulados y 5 axiomas para deducir, a partir de
ah, el corpus de su geometra.
Incluimos un apendice a este captulo con los postulados y axiomas de Euclides as como
los axiomas de Hilbert para la geometra euclideana.
1
y, en ultima instancia, remitir las indagaciones sobre estos principios a una parte que podramos llamar
de fundamentos de la teora.
2
Euclides de Alejandra, ca 300 A.C.
3
o
4
Sir Thomas L. Heath escribe en la introduccion de la reimpresi on de 1932 de los elementos (No 891 in
the Everymans Library): The simple truth is that it was not written for schoolboys or schoolgirls, but for
the grown man who would have the necessary knowledge and judgment to appreciate the highly contentious
matters which have to be grappled with in any attempt to set out the essentials of Euclidean geometry as a
strictly logical system, and, in particular, the diculty of making the best selection of unproved postulates or
axioms to form the foundation of the subject....
1
2 Rodolfo San Agustn Chi
As, consideremos primeramente el
Quinto postulado de Euclides. Si una recta incidente con dos rectas hace angulos in-
ternos y de la misma parte menores que dos rectos, prolongadas esas dos rectas al innito
coincidir an por la parte en que esten los angulos menores que dos rectos.

Figura 1.1: + <


Vemos directamente que este postulado no es ni tan simple ni tan evidente como
los dem as. En efecto, por mucho tiempo se trabajo, en diferentes direcciones, sobre las
relaciones de este con los dem as postulados de Euclides.
Particularmente,
1. Surgieron varias formulaciones equivalentes a el.
2. Se trat o (in utilmente, por cierto) de obtener este postulado como una consecuencia
de los dem as.
3. Se trat o (y se logr o) probar su independencia respecto de los dem as.
4. En diversas ramas de la geometra se le sustituy o por algunas de sus negaciones
logicas
5
.
Respecto del primer punto anterior tenemos el famoso teorema de los angulos internos
de un tri angulo :
Postulado V (i). Los angulos internos de un tri angulo suman dos rectos.
Con esta versi on del quinto postulado sabemos que basta tener dos pares de angulos
correspondientes congruentes, en dos tri angulos cualesquiera, para que los terceros pares de
angulos tambien sean congruentes entre s. As aplicaremos este razonamiento, sin mayor
argumentacion, en la prueba del teorema (1.1.0.3).
Otras formulaciones de uso frecuente del quinto postulado son las siguientes:
Postulado V (ii). [Playfair
6

7
] Por un punto dado no incidente con una recta tambien
dada solo se puede trazar una paralela a ella.
Postulado V (iii). Los angulos alternos internos entre paralelas son iguales.
3 Notas de Geometra Moderna I

Figura 1.2:
Postulado V (iv). Un angulo exterior de un tri angulo es igual a la suma de los dos interiores
no adyacentes a el.


Figura 1.3: = +.
En el libro ([Eve69], p. 320 y siguientes, incluyendo ejercicios) hay m as equivalencias
del quinto postulado junto con una excelente discusion sobre esta labor
8
.
1.1. Congruencia de triangulos
Siguiendo a Hilbert [AS80], el smbolo denotara congruencia tanto de segmentos
como de angulos.
En principio, podriamos considerar que la nocion de congruencia se reere a guras
iguales; es decir, a guras que tienen la misma forma y el mismo tama no. Uno podria,
incluso, iniciar una teora de guras congruentes considerando las guras m as simples en el
plano: los tri angulos.
Para ello, recordemos que Hilbert (cf. [AS80]) declara la congruencia entre segmentos
(y angulos) como una relaci on indenida y la reglamenta mediante los axiomas correspon-
dientes. Nos resultara particularmente util el axioma III.5, el cual establece una relaci on
entre ciertos objetos unidimensionales, los segmentos de recta y los angulos, con objetos
bidimensionales, los tri angulos:
5
Lo cual dio lugar a las geometras no euclideanas.
6
John Playfair, 1748-1819. Es curioso que el nombre de Playfair este asociado a este axioma, dado que el
mismo se nala que lo obtuvo de Proclo.
7
Proclo, 410-485 D.C.
8
Tambien las paginas electronicas http://www.cut-the-knot.org/triangle/pythpar/Attempts.shtml y
http://www.beva.org/math323/asgn6/nov19.htm, entre otras, dan muy buena cuenta de ello.
4 Rodolfo San Agustn Chi
Axioma III.5. En dos tri angulos ABC y A

, si
AB A

,
AC A

y
BAC B

entonces ACB A

.
C
A
B C B
A
Figura 1.4: Axioma III.5 de Hilbert
As, damos la denicion que sigue:
Denicion 1.1.0.1. Dos tri angulos son congruentes si sus lados correspondientes son con-
gruentes y sus angulos correspondientes tambien son congruentes.
De acuerdo a esta denicion, extendemos el signicado del smbolo para parejas de
tri angulos en esta relaci on.
A continuaci on tenemos los tres teoremas b asicos de la teora de congruencia de tri angu-
los. En el texto [LMdlESA77] hay una presentacion de ellos basada en problemas topogr a-
cos la cual, tal como muchos textos elementales de geometra, emplea el llamado metodo o
principio de superposici on.
9
Como ya hemos mencionado en el prefacio, en lugar de esto,
nosotros probaremos estos teoremas empleando b asicamente los axiomas de congruencia de
Hilbert.
Teorema 1.1.0.2 (ALA). Si
(1) CBA C

,
(2) BC B

y
(3) ACB A

Entonces
ABC A

.
Demostraci on. Basta vericar que AB A

.
En la gura (1.5), con las hip otesis (1), (2) y (3) :
9
Este metodo consiste en la colocaci on ideal de un objeto en la posicion de otro para demostrar que los
dos coinciden. Euclides solamente emple o este procedimiento en I.4, I.8 y III.24, aunque pudo usarlo en
muchas otras proposiciones tambien.
5 Notas de Geometra Moderna I
C
C B
A
A
A
B
Figura 1.5:
(4) Sea A

AB A

axioma III.1.
(5) BCA B

axioma III.5 en (2), (1) y (4).


(6) B

de (3) y (5).
(7)

axioma III.4 en (6).


(8) A

= A

(7)
Teorema 1.1.0.3 (LAL).
10
Si
(1) AC A

,
(2) ACB A

y
(3) CB C

.
Entonces
ABC A

.
Demostraci on. En la gura (1.6), con las hip otesis (1), (2) y (3) :
B C
C B
A A
Figura 1.6:
(4) BAC B

axioma III.5 en (1), (2) y (3).


(5) ABC A

teorema (ALA) en (1), (2) y (4)


10
De los tres teoremas de congruencia, este es el que mas uso Euclides, incluso en la proposicion XI.4 y
muchas otras del libro XI, donde los tri angulos estan en planos diferentes.
6 Rodolfo San Agustn Chi
Observaci on 1.1.0.4. La denominacion LAL para este teorema se reere a que, en las
hip otesis, el angulo de referencia es precisamente el incluido entre los otros dos lados. En
otras palabras, un teorema ALL, por ejemplo, no es posible: En la gura 1.7, Z es el centro
de una circunferencia que pasa por Y y Y

; as, los tri angulos XY Z y XY

Z satisfacen las
hip otesis ALL pero no son congruentes. En el teorema ALA (1.1.0.2), por la versi on (i) del
quinto postulado, esta distincion resulta inmaterial.
X
Y
Y
Z
Figura 1.7: Contraejemplo a un supuesto teorema ALL.
Damos a continuaci on el teorema I.5 de Euclides. Se le conoce como Pons Asinorum
(PA) pues, supuestamente, es difcil hacer que un burro pase por un puente
11
o, probable-
mente, por la gura que aparece en la complicada prueba de Euclides mismo. Posteriormente
(ca. 340 D.C.) Pappus
12
di o una prueba m as simple. En 1956, un programa de Marvin Min-
sky encontr o la demostracion que damos a continuaci on. Curiosamente, dicha demostracion
sigue la lineas de la demostracion de Pappus. En ambos casos, la idea es considerar al
tri angulo dado, ABC, con AB AC, digamos, como si se tratase de los dos tri angulos
ABC y ACB. Posteriormente, Herbert Gelernter (1960) publico una prueba similar real-
izada con un demostrador autom atico de teoremas de geometra basado en el programa de
Minsky.
Teorema 1.1.0.5 (PA). En un tri angulo cualquiera los angulos opuestos a lados congru-
entes son congruentes.
Demostraci on. Sea ABC un tri angulo tal que
(1) AB AC. Hipotesis
Entonces
(2) ABC A

axioma III.5 en ABC y ACB


Tambien se tiene el teorema recproco y cuya demostracion queda como ejercicio:
11
De la Wikipaedia: It takes its name as the rst real test in the Elements of the intelligence of the reader
and as a bridge to the harder propositions that followed. Its location in that text is much more advanced
than where the problem is posed in present-day geometry textbooks for high-school students.
12
Papus de Alejandra; vivio entre los a nos 200 y 350 D.C. The Hutchinson dictionary of scientic
biography. Abingdon, Oxon: Helicon Publishing, 895.
7 Notas de Geometra Moderna I
A
C B C
A
B
Figura 1.8:
Teorema 1.1.0.6. En un tri angulo cualquiera los lados opuestos a angulos congruentes son
congruentes.
Com unmente se consideran positivamente orientados aquellos angulos que se describen
en contra de las manecillas del reloj (y, consecuentemente, los angulos negativamente orien-
tados se describen en el sentido favorable a las manecillas de dicho instrumento). As, quedan
orientados coherentemente todos los angulos del plano euclideano y, con ello, tenemos una
aritmetica de angulos en la cual usamos la operaci on siguente:
se suman angulos con la misma orientacion
y se restan los angulos con orientaciones opuestas.
Podramos haber orientado todos los angulos al reves y, entonces, la operaci on mencionada
tendra exactamente la misma funcionalidad que en el primer caso. Es por esto que no
nos jaremos tanto en la orientacion efectiva de cada uno de nuestros angulos sino en la
orientacion relativa de los pares de angulos en el plano (i.e. si dos angulos dados tienen la
misma orientacion o bien tienen orientaciones opuestas).
Teorema 1.1.0.7 (LLL). Si
(1) AB A

,
(2) BC B

y
(3) CA C

Entonces
ABC A

.
Demostraci on. Por simetra, basta probar, por ejemplo, que CAB C

.
As, en la gura (1.9), con las hip otesis (1), (2) y (3), sea X un punto tal que
8 Rodolfo San Agustn Chi
B C
C B
A A
X
Figura 1.9:
(4) C

X CBA y axioma III.4.


(5) B

X BA. axioma III.1.


(6) A

XB

X teorema PA (1.1.0.5) en (1) y (5).


(7) B

XC

BAC teorema LAL (1.1.0.3) en (2), (4) y (5).


(8) XC

AC (7).
(9) XC

(3) y (8).
(10) C

XA

XA

teorema PA (1.1.0.5) en (9).


(11) C

XB

suma de angulos en (6) y (10).


(12) C

XB

CAB (7).
(13) CAB C

(11) y (12).
Nota.- Para el paso (7) de esta demostracion hubiese bastado usar el axioma III.5 de Hilbert
pero realmente necesitamos la conclusion del teorema LAL con toda su fuerza para los pasos
(7), (8), (9), (10) y (12).
Observaci on 1.1.0.8. Los teoremas de congruencia de tri angulos justican el principio de
superposici on.
En efecto, el cuarto postulado de Euclides (cosas que coinciden entre s son iguales)
se puede ver como un intento de levantar y mover una gura para superimponerla a otra
para, as, vericar la congruencia de ambas. Esto nos lleva directamente a pensar en las
transformaciones rgidas. Felix Klein
13
desarrollo estas ideas en su programa de Erlangen, el
cual consiste, b asicamente, en el estudio de las diversas geometras a traves de los invariantes
de sus grupos de transformaciones.
13
Felix Klein(1849 - 1925). En 1872, tras ingresar como profesor en la Universidad de Erlangen, dio una
conferencia inaugural en la que propuso el estudio de la geometra desde el punto de vista de la teora de
grupos, lo cual se conoce como el programa de Erlangen. En 1895 impuls o el proyecto de la Encyklopadie
der Mathematischen Wissenschaften, la cual supervis o hasta su muerte. Tambien destaco por sus iniciativas
en pro de la reforma educativa en Alemania.
9 Notas de Geometra Moderna I
1.2. El teorema de Pitagoras
Teorema 1.2.0.9. En un tri angulo rect angulo de lados a, b y c. Si el lado c esta opuesto
al angulo recto, entonces
a
2
+b
2
= c
2
Demostraci on.
14
En cada lado del cuadrado exterior en la gura 1.10 marcamos un punto
que lo divida en segmentos de longitudes a y b, respectivamente. Esta demostracion se basa
en calcular el area de la gura de dos maneras diferentes.
a
a
b
b
a
b
a b
c
c
c
c
Figura 1.10: (a +b)
2
= 2ab +c
2
(1)

Area de la gura completa = (a +b)
2
= a
2
+ 2ab +b
2
(2) Los cuatro tri angulos son todos congruentes entre s teorema (LAL)
(3)

Area de los 4 tri angulos = 2ab (2).
(4) La gura central es un cuadrado con lado de longitud c (2).
(5)

Area de la gura central = c
2
(4).
(6)

Area de la gura completa = 2ab +c
2
(3) y (5).
(7) a
2
+b
2
= c
2
(1) y (6).
1.3. Cuadrilateros cclicos
1.3.1.

Angulos inscritos en una circunferencia
El angulo inscrito en una circunferencia es aquel que tiene su vertice en dicha cir-
cunferencia. El angulo central es el que tiene su vertice en el centro de una circunferencia
dada. Los lados de cada uno de dichos angulos abarcan o subtienden cierto arco de la
circunferencia. Por ejemplo, en la gura 1.11, el angulo AOC abarca el arco AC de la
circunferencia que no pasa por B.
14
Esta prueba es similar a las que utilizan cuatro tri angulos en la pagina electronica http://www.cut-
the-knot.org/pythagoras/ donde se encuentran muchas referencias y, hasta ahora, 98 pruebas de este mismo
teorema.
10 Rodolfo San Agustn Chi
Teorema 1.3.1.1. El angulo inscrito en una circunferencia es la mitad del angulo central.
B
D
A
C
O
A
C
B
D O
Figura 1.11:
Demostraci on. En la gura (1.11), sea ABC un angulo inscrito y sea O el centro de la
circunferencia. Consideramos el di ametro BD.
Entonces, el tri angulo ABO es is osceles y, por lo tanto,
2ABD +BOA = = BOA+AOD
y, de aqu,
ABD =
1
2
AOD
Analogamente,
DBC =
1
2
DOC
y, de ambas identidades, tenemos el resultado deseado.
Corolario 1.3.1.2.
1.

Angulos que subtienden el mismo arco son iguales y angulos que subtienden arcos
suplementarios son suplementarios.
2. En la gura (1.12), OP es tangente a la circunferencia.
Entonces,
OPC PDC.
11 Notas de Geometra Moderna I
P
O
C
D
Figura 1.12:
1.3.2. Cuadrilateros cclicos
Consideremos un conjunto, digamos c de cuatro puntos en el plano. Estos puntos
determinan un cuadrilatero una vez que se les ha ordenado cclicamente Los lados del
cuadrilatero son los segmentos de recta determinados por vertices consecutivos ante este
ordenamiento. Dos vertices que est an unidos por un lado del cuadrilatero se llaman adya-
centes. Analogamente, dos lados que comparten un vertice son adyacentes. Dos vertices
que no comparten un mismo lado del cuadrilatero son opuestos; en este caso, el segmento
de recta que ellos determinanlas es una diagonal del cuadrilatero. Tambien, dos lados que
no comparten un vertice son opuestos. Un cuadrilatero es cruzado si dos lados opuestos
se cortan.
As,
Ya que hay tres formas de ordenar cclicamente a los elementos de un conjunto con
cuatro elementos, cada conjunto c determina tres cuadrilateros distintos (los cuales
comparten el mismo conjunto de vertices).
Los seis segmentos de recta que unen a dichos puntos se distribuyen en tres pares de
segmentos opuestos y, entonces, cada cuadrilatero tiene dos pares de lados opuestos y
un par de diagonales.
Los papeles de par de lados opuestos y par de diagonales se van intercambiando de
acuerdo al orden cclico que se vaya dando a los puntos del conjunto c.
Denicion 1.3.2.1. Un cuadrilatero es cclico si sus vertices est an en una circunferencia.
Primeramente vemos que
Lema 1.3.2.2. Un cuadril atero cclico solamente puede ser cruzado o convexo.
Demostraci on. Sean A, B y C vertices consecutivos del cuadrilatero. Ellos parten a la
circunferencia en los tres arcos AB, BC y CA.
Si D, el cuarto vertice del cuadrilatero, est a en el arco AB, entonces los vertices C y
D est an en arcos distintos AB de la circunferencia y de aqu que los segmentos AB y CD
se corten.
12 Rodolfo San Agustn Chi
Analogamente, si D est a en el arco BC los segmentos BC y AD se cortan.
Finalmente: si D est a en el arco CA, ninguno de los segmentos CD y DA corta a
ninguno de los segmento AB y BC. Este es el caso del cuadrilatero convexo.
Del inciso 1 del corolario 1.3.1.2 tenemos el siguiente resultado clasico:
Corolario 1.3.2.3. En un cuadril atero cclico los pares de angulos opuestos son iguales o
suplementarios.
As, por el lema 1.3.2.2, el caso cruzado corresponde al de angulos opuestos iguales.
El resultado recproco tambien es verdadero:
Corolario 1.3.2.4. Un cuadril atero convexo o cruzado es cclico si sus angulos opuestos
son iguales o suplementarios.
Nuevamente, por el lema 1.3.2.2, el caso cruzado corresponde al de angulos opuestos
iguales.
Demostraci on.
1er. caso
(1) ABCD es un cuadrilatero convexo tal que
(2) ABC +CDA = .
(3) Sea D

el otro punto de intersecci on de la recta AD con la circunferencia


que pasa por los puntos A, B y C.
(4.1) Si ABCD

es convexo (i.e., si D

AD), entonces,
por el corolario 1.3.2.3, CD

A+ABC = .
y CDA CD

A.
(4.2) Si ABCD

es cruzado,
por el corolario 1.3.2.3, ABC AD

C
y CDA+CD

A = .
(5) As, en cualquiera de los dos subcasos (4.1 y 4.2), en CDD

, D = D

B
A
D
D
C
D
B
A
D
C
Figura 1.13: Los dos subcasos cuando ABCD es convexo.
13 Notas de Geometra Moderna I
2o. caso
La demostracion de este caso es muy similar al 1er. caso. Se deja como ejercicio.
Lema 1.3.2.5 (Ley de los senos). En un tri angulo ABC,
AB
senACB
=
BC
senBAC
=
CA
senCBA
(1.1)
Demostraci on. En la gura (1.14), sea D el pie de la altura por el vertice A.
A
B D C
Figura 1.14:
Por denicion del seno de un angulo,
senCBA =
DA
BA
y senACB =
AD
CA
as,
AB senCBA = CA senACB
y, de aqu, la armacion.
Una consecuencia inmediata es el teorema de la bisectriz:
Teorema 1.3.2.6 (de la bisectriz). Si la bisectriz del angulo BAC corta en un punto L a
la recta BC, entonces
BL
LC
=
BA
AC
. (1.2)
1.4. Segmentos de recta dirigidos
De acuerdo a Eves [Eve69]: Una de las inovaciones de la geometra moderna elemental
es el empleo, cuando es util, de magnitudes con sentido, [...]. La extencion del sistema
numerico a la inclusion de n umeros tanto positivos como negativos fue lo que condujo a
14 Rodolfo San Agustn Chi
A
B C L
Figura 1.15:
este paso [...] en la geometra
15
. As, quedaremos plenamente instalado en la geometra
moderna al considerar segmentos dirigidos sobre una recta.
Sin embargo, tal y como en el caso de los angulos, nuestras consideraciones al respecto
tendran que ver mayormente con la orientacion relativa de dos segmentos sobre una misma
recta. Es decir, nos interesamos por saber si dos segmentos en la misma recta tienen o no
la misma orientacion.
El hecho de que AB y BA sean iguales en magnitud pero de sentido contrario se indica
con la ecuaci on que sigue:
AB = BA
o, equivalentemente,
AB +BA = 0. (1.3)
Es decir, de acuerdo al axioma III.1 de Hilbert, dado el segmento AB, si tomamos
a = a

, A

= B y el lado de A desde el punto B, entonces B

= A.
La formula de Chasles
16
expresa de forma muy simetrica la aditividad de segmentos.
Dicha aditividad se consigna en el axioma III.3 de Hilbert:
Axioma III.3. Sean AB, BC dos segmentos sin puntos comunes excepto por B sobre la
recta a. Ademas sean A

, B

dos segmentos sobre la misma o sobre otra distinta a

igualmente sin puntos comunes excepto por B

. En ese caso, si
AB A

y BC B

entonces
AC A

.
Teorema 1.4.0.7 (Formula de Chasles). Si A, B y C son tres puntos alineados, entonces
AB +BC +CA = 0 (1.4)
15
... aunque A. Girard, R. Descartes y otros introdujeron en la geometra [el empleo de] segmentos negativos
durante el siglo XVII la idea de las magnitudes con sentido fue explotada sistematicamente por primera vez
a principios del siglo XIX por Lazare Nicolas Marguerite Carnot (1753-1823. Geometrie de position, 1803)
y August Ferdinand Mobius (1790-1868. Der barycentrische Calk ul, 1827.) .
16
Michel Chasles, 1793-1880.
15 Notas de Geometra Moderna I
Por supuesto, el caso A = C arroja que AA = 0.
Emplearemos algunas veces la reformulaci on siguiente del teorema 1.4.0.7:
Corolario 1.4.0.8. Si O es cualquier punto sobre la recta AB, entonces
AB = OB OA. (1.5)
Teorema 1.4.0.9 (Formula de Euler
17
). Si A, B, C y D son cuatro puntos alineados,
entonces
AB CD +AC DB +AD BC = 0 (1.6)
Esta formula se puede probar utilizando la formula (1.5) para escribir el termino de la
izquierda como sigue:
(DB DA) CD + (DC DA) DB + (DC DB) AD
posteriormente, desarrollando y simplicando, se concluye.
1.4.1. Razon de particion de un segmento dirigido
Denicion 1.4.1.1. Si A, B y P son tres puntos (no necesariamente distintos) alineados,
AP
PB
es la raz on en que el punto P divide al segmento orientado AB.
Teorema 1.4.1.2. Dada una razon r (,= 1) hay exactamente un punto que divide a un
segmento dado en dicha razon.
Demostraci on.
Unicidad: Sea AB el segmento orientado dado (A ,= B).
(1)
AP
PB
=
AQ
QB
= AP QB = AQ PB
(2) AP QB +AQ BP +AB PQ = 0 por la formula de Euler (1.6).
(3) AB PQ = 0 de (1) y (2).
(4) PQ = 0 pues AB ,= 0 al ser A ,= B.
(5) P = Q
Existencia: Sea x := AP.
(1) PB = AB x
(2)
AP
PB
=
x
AB x
(= r)
(3) y de aqu que x =
r AB
1 +r
17
Leonhard Euler, 1707-1783.
16 Rodolfo San Agustn Chi
Es decir, P es el punto tal que
1. Esta a una distancia x de A y
2. P est a entre A y B r > 0.
Por otra parte, si denimos:
AP
PB
:=

si P = B,
1 si P = .
tendremos una funcion biyectiva (ver la gr aca 1.16):

AB R
P
AP
PB
.
A B
Figura 1.16: La funcion P
AP
PB
, con [ AB [= 1.
Teorema 1.4.1.3 (generalizado de la bisectriz). Si el vertice A del tri angulo ABC esta unido
a cualquier punto L de la recta BC, entonces
BL
LC
=
AB senBAL
CA senLAC
.
17 Notas de Geometra Moderna I
Demostraci on. En el ABL de la gura (1.17), por el lema (1.3.2.5) tenemos que
AB
senALB
=
BL
senBAL
.
Y an alogamente en el ALC:
LC
senLAC
=
CA
senCLA
A
L L C B
Figura 1.17: Teorema generalizado de la bisectriz.
Ya que los angulos ALB y CLA son iguales o suplementarios (dependiendo esto
ultimo de que el punto L este fuera o dentro del segmento BC), tenemos que
senALB = senCLA
y, de aqu, la armacion
Ejercicios
1. Probar el recproco de Pons asinorum (Teorema 1.1.0.6): En un tri angulo cualquiera
los lados opuestos a angulos congruentes son congruentes.
2. Probar que la mediatriz de un segmento AB (A ,= B) es el lugar equidistante de A y
B. Esto es, se trata del lugar geometrico siguiente:
X[AX BX
3. Probar el recproco del teorema de Pit agoras (1.2.0.9): En un tri angulo de lados a, b
y c, si
a
2
+b
2
= c
2
entonces, el angulo opuesto al lado c es un angulo recto.
4. Probar el corolario (1.12): En la gura (1.12), si OP es tangente a la circunferencia,
entonces,
OPC PDC.
18 Rodolfo San Agustn Chi
C
D
A
B
D
D
A
C
B
D
Figura 1.18: Los dos subcasos cuando ABCD es cruzado.
5. Demostrar el segundo caso del corolario 1.3.2.4:
Un cuadrilatero cruzado ABCD tal que ABC CDA, es cclico (ver la gura 1.18).
6. A partir de la ley de los senos, demostrar el teorema de la bisectriz (1.3.2.6):
Si la bisectriz del angulo BAC corta en un punto L a la recta BC, entonces
BL
LC
=
BA
AC
.
7. Las rectas de dos haces con vertices diferentes est an en correspondencia biunvoca de
tal manera que las intersecciones de rectas correspondientes son colineales. Encontrar
un par de rectas perpendiculares en el primero, para el cual las rectas correspondientes
en el otro, sean tambien perpendiculares.
8. Haz una construccion geometrica para los puntos que dividen a un segmento de recta
dado en las razones
a
b
y
a
b
(a y b positivos).
9. Prueba el teorema de Stewart
18
:
Si A, B y C son tres puntos colineales y D es cualquier cuarto punto, entonces
DA
2
BC +DB
2
CA+DC
2
AB +AB BC CA = 0.
Sugerencia. Considera primero el caso en que D est a en la misma recta. Despues
dibuja una perpendicular desde D a la recta.
10. Si A, B y C son puntos colineales y si P, Q y R son los puntos medios de BC, CA y
AB respectivamente, demuestra que el punto medio de CR coincide con el de PQ.
18
Matthew Stewart (1717-1785) enunci o este teorema, sin demostraci on, en 1746; Thomas Simson (1710-
1761) en 1751, Leonhard Euler en 1780 y L.N.M. Carnot en 1803 lo redescubrieron y volvieron a probar.
El caso en que D esta en la misma recta se trata en el libro Synagoge (colecci on, tratado o antologia
matematica)(ca. 340) de Pappus.
19 Notas de Geometra Moderna I
1.5. Apendice
1.5.1. Principios de Euclides
Euclides recopil o en los elementos, en el siglo III a. C., el conocimiento que hasta
entonces se tena sobre la geometra
19
. No se trata solamente de un recopilaci on sino de
un desarrollo sistematico de aquellos conocimientos a partir de un reducido conjunto de
principios, los cuales se expresan en las deniciones, nociones comunes (o axiomas) y pos-
tulados. Los primeros denen los objetos de la geometra: punto, recta, angulo, etc., las
nociones comunes establecen algunas reglas muy generales de operaci on y en los postulados
se establecen las relaciones fundamentales entre los objetos de la geometra.
Una traduccion casi literal de los postulados y axiomas de Euclides (ver [Euc92]):
Post ulese:
1. Trazar una lnea recta desde un punto cualquiera a otro punto cualquiera.
2. Prolongar por continuidad en lnea recta una lnea recta delimitada.
3. Para cada centro y radio describir un crculo.
4. Que todos los angulos rectos son iguales entre s.
5. Que, si una recta incidente con dos rectas hace angulos internos y de la misma parte
menores que dos rectos, prolongadas esas dos rectas al innito coincidiran por la parte
en que esten los angulos menores que dos rectos.
Axiomas o nociones comunes
1. Las cosas que son iguales a la misma cosa tambien son iguales entre s.
2. Si a cantidades iguales se suman otras tambien iguales, los totales ser an iguales.
3. Si se restan cantidades iguales de otras tambien iguales, los residuos ser an iguales.
4. Las cosas que coinciden entre s son iguales entre s.
5. El todo es mayor que una parte.
19
Contrario a las impresiones mas generalizadas, muchas de las proposiciones (de un total de 465) de
Euclides tratan, no de geometra, sino de teora de los n umeros y de algebra (ver [Eve69], pp. 19 y sigs.).
20 Rodolfo San Agustn Chi
1.5.2. Axiomas de Hilbert
Se consideran tres conjuntos distintos de objetos: puntos, rectas y planos.
Se considera que dichos objetos guardan entre s las relaciones que llamaremos estar en,
estar entre, ser congruente. La descripcion precisa y matematicamente completa de estas
relaciones se sigue de los axiomas de la geometra.
Primer grupo: axiomas de incidencia.
Los axiomas de este grupo establecen una relaci on incidencia entre los puntos-rectas-
planos como sigue:
I.1 Para cada dos puntos A y B hay alguna recta a que contiene a cada uno de los puntos
A y B.
I.2 Para cada dos puntos A y B no hay m as que una recta que contiene a cada uno de los
puntos A y B.
I.3 Hay al menos dos puntos en cada recta.
Hay al menos tres puntos que no est an en la misma recta.
I.4 Para cada tres puntos A, B y C que no esten en la misma recta hay un plano que
contiene a cada uno de los puntos A, B y C.
Cada plano contiene alg un punto.
I.5 Para tres puntos A, B y C que no esten en una y la misma recta no hay m as que un
plano que contiene a cada uno de los puntos A, B y C.
I.6 Si dos puntos A y B de alguna recta a est an en alg un plano entonces cada punto de
a est a en el plano .
I.7 Si dos planos y tienen un punto com un A entonces tienen al menos otro punto
com un B.
I.8 Hay al menos cuatro puntos que no est an en un [mismo] plano.
Segundo grupo: axiomas de orden.
Estos axiomas denen el concepto de estar entre, para los puntos de una recta.
II.1 Si B est a entre A y C, entonces A, B y C son tres puntos distintos de una recta y
tambien estar a B entre C y A.
II.2 Para dos puntos A y C siempre hay al menos un punto B en la recta AC tal que C
est a entre A y B.
II.3 De entre tres puntos de una recta no hay m as que uno que est a entre los otros dos.
21 Notas de Geometra Moderna I
II.4 Sean A, B y C tres puntos no colocados en lnea recta y sea a una recta en el plano
ABC, que no pasa por ninguno de los puntos A, B, C. Si la recta a pasa por alg un
punto [interior] del segmento AB, pasara tambien por alg un punto del segmento AC
o bien, por alg un punto del segmento BC.
Tercer grupo: axiomas de congruencia.
III.1 Si A y B son dos puntos en alguna recta a y A

es un punto en la misma o en otra


recta a

entonces siempre es posible hallar un punto B

en un lado dado de la recta a

tal que el segmento AB sea congruente con el segmento A

.
En smbolos
AB A

III.2 Si un segmento A

y alg un otro segmento A

son congruentes a el mismo seg-


mento AB entonces el segmento A

tambien es congruente al segmento A

,
O, brevemente, si dos segmentos son congruentes a un tercero son congruentes entre
s.
III.3 Sean AB, BC dos segmentos sin puntos comunes excepto por B sobre la recta a.
Ademas sean A

, B

dos segmentos sobre la misma o sobre otra distinta a

igual-
mente sin puntos comunes excepto por B

. En ese caso, si
AB A

y BC B

entonces
AC A

.
Este axioma expresa el requerimiento de aditividad de segmentos.
III.4 Sean (h, k) un angulo en un plano y a

una recta en un plano

; se ja un lado de
a

en

. Sea h

un rayo de la recta a

el cual parte del punto O

. Entonces, en el plano

, hay uno y solo un rayo k

tal que el (h, k) es congruente al (h

, k

) y a la vez que
todos los puntos interiores del (h

, k

) est an en el lado dado de

. Simb olicamente:
(h, k) (h

, k

).
Todo angulo es congruente consigo mismo, esto es: vale siempre
(h, k) (h, k).
III.5 En dos tri angulos ABC y A

, si
22 Rodolfo San Agustn Chi
AB A

,
AC A

y
BAC B

entonces ABC A

.
Cuarto grupo: axioma de paralelismo
IV.1 (Axioma de Euclides) Sea a una recta y A un punto fuera ella. Entonces hay a lo m as
una recta en el plano determinado por a y A que pasa por A y no intersecta a a.
Quinto grupo: axiomas de continuidad
V.1 (Axioma de Arqumedes) Si AB y CD son dos segmentos hay un n umero [natural]
n tal que n segmentos CD construidos consecutivamente desde A a lo largo del rayo
desde A por B pasaran m as alla del punto B.
V.2 (Axioma de la completez de la recta) Una extencion de un conjunto de puntos en una
recta con sus relaciones de orden y congruencia que preserve las relaciones entre los
elementos originales as como las propiedades fundamentales de orden y congruencia
en una recta que se siguen de los grupos de axiomas I-III y V.1 es imposible.
Captulo 2
Semejanza
Intuitivamente: dos guras son semejantes si tienen la misma forma. En el caso particu-
lar de tri angulos, tener la misma forma solamente requiere que los angulos correspondientes
sean iguales. Es decir, la relaci on de semejanza se reere a guras que teniendo la misma
forma pudieran tener tama nos diferentes.
Tal como en el caso de congruencia, podemos iniciar la teora estudiando las guras
bidimensionales m as simples: los tri angulos.
2.1. Semejanza de triangulos.
Podemos considerar al lema siguiente como la base de la teora de la proporcionalidad
de guras. A partir de este lema, tendremos que los tri angulos semejantes son aquellos
que tienen sus angulos correspondientes congruentes y sus lados correspondientes propor-
cionales.
Lema 2.1.0.1. Una recta paralela a uno de los lados de un tri angulo divide proporcional-
mente a los otros dos y recprocamente.
Demostraci on. En un tri angulo ABC:
A
X
B C
B C
Y
Figura 2.1:
23
24 Rodolfo San Agustn Chi
(1) Sea B

| BC Hipotesis.
(2)

Sea X el pie de la perpendicular desde C

hacia la recta AB.


Hipotesis.
(3)

Sea Y el pie de la perpendicular desde B

hacia la recta AC.


Hipotesis.
As,
(4)
(AC

)
(B

B)
=
1
2
AB

XC

1
2
B

B XC

=
AB

XC

BB

por (1) y (2) y


la formula 1.
(5)
(AC

)
(B

C)
=
1
2
AC

Y B

1
2
C

C Y B

=
AC

Y B

CC

por (1) y (3) y


la formula 1.
(6) (B

C) = (B

B)

Ambos tri angulos tienen la


misma base y la misma altura.
(7)
AC

C
=
AB

B
(4), (5) y (6).
(8)
AC
CC

=
AB
BB

an alogo a (7).
(9)
AC
AC

=
AB
AB

(7) y (8).
Recprocamente: Ahora supongamos que
(10)
AC
AC

=
AB
AB

Hipotesis.
(11) B

D | BC Hipotesis.
As,
(12)
AB
AB

=
AC
AD
(11) y () en BAC y B

AD

.
(13)
AC
AC

=
AC
AD
(10) y (12).
(14)
AD +DC
AD
=
AC

+C

C
AC

(13).
(15)
CD
DA
=
CC

A
(14).
(16) D = C

(15).
Denicion 2.1.0.2. Dos tri angulos son semejantes si
(i) Sus angulos correspondientes son congruentes y
(ii) Sus lados correspondientes son proporcionales.
Notaci on 2.1.0.3. ABC A

signica que los tri angulos ABC y A

son
semejantes.
25 Notas de Geometra Moderna I
En general,
Denicion 2.1.0.4. Dos polgonos con el mismo n umero de lados son semejantes si
(i) Sus angulos correspondientes son congruentes y
(ii) Sus lados correspondientes son proporcionales.
Es decir, para el caso de tri angulos, para tener la semejanza de dichas guras, tenemos
seis condiciones a vericar. Y, en general, para el caso de polgonos con n lados ser an 2n
las condiciones a satisfacer.
Tal como en el caso de congruencia, nuevamente, tenemos tres resultados estructurales
(los teoremas 2.1.0.5, 2.1.0.6 y el inciso (e) del corolario 2.1.0.7) para esta teora,
Dichos teoremas nos dicen, brevemente, que las seis condiciones de semejanza de
tri angulos son redundantes; que, en realidad, ciertas ternas de estas condiciones implican
las dem as.
Teorema 2.1.0.5. Dos tri angulos que tienen sus angulos correspondientes iguales tienen
lados correspondientes proporcionales.
Demostraci on. Sean ABC y A

tales que
A
B
A
B C
A
B C
Figura 2.2:
(1) =

Hipotesis.
(2) =

Hipotesis.
(3) =

Hipotesis.
(4) Sea B

CB tal que B

C B

axioma III.1.
(5) Sea A

CA tal que A

C A

axioma III.1.
(6) A

C axioma III.5 en (3), (4) y (5).


(7) A

| AB (2) y (6)
(8)
CB
CB

=
CA
CA

lema 2.1.0.1 en (7).


(9)
CB
C

=
CA
C

(4), (5) y (8).


(10)
CA
C

=
BA
B

an alogamente
26 Rodolfo San Agustn Chi
Teorema 2.1.0.6. Si
AB
A

=
BC
B

=
CA
C

entonces

.
Demostraci on.
C
B
C
B C
A
B
A
Figura 2.3:
(1)
AB
A

=
BC
B

=
CA
C

Hipotesis.
(2) Sea B

tal que AB A

axioma III.1.
(3) Sea C

tal que AC A

axioma III.1.
(4)
A

=
A

(1), (2), (3).


(5) B

| B

lema 2.1.0.1 en (4).


(6) C

y A

(5).
(7)
A

=
A

=
C

teorema 2.1.0.5 en (6) para A

y AB

.
(8)
BC
B

=
B

(1) y (7).
(9) BC B

(8).
(10) ABC A

teorema (LLL), (2), (3) y (9).


(11) BAC B

(10).
(12) BAC B

(6) y (11).
Corolario 2.1.0.7. Las proposiciones que siguen son equivalentes:
Dos tri angulos
(a) Son semejantes.
(b) Sus angulos correspondientes son congruentes.
(c) Sus lados correspondientes son proporcionales.
(d) Dos pares de angulos correspondientes son congruentes.
(e) Dos pares de lados correspondientes son proporcionales y los angulos incluidos son
congruentes.
27 Notas de Geometra Moderna I
Demostraci on.
Ya hemos visto que las condiciones (a), (b), (c) y (d) son equivalentes. Tambien se
tiene que (a) = (e). As, bastara probar que (e) = ( ).
Observaci on 2.1.0.8. En la gura (1.12), ya que OPC PDC, los tri angulos OPC
y ODP son semejantes.
El particular,
OP
2
= OC OD (2.1)
Corolario 2.1.0.9 (Teorema de Thales). Si tres (o mas) paralelas son cortadas por dos
transversales los segmentos de una recta son proporcionales a los segmentos correspondientes
de la otra.
Corolario 2.1.0.10. Si dos pares de lados correspondientes de tri angulos rect angulos son
proporcionales, entonces los tri angulo son semejantes.
Notese que siempre es posible orientar polgonos. Esto, adem as, se puede hacer sola-
mente de dos maneras diferentes. Ademas, esto ocurre de forma tal que siempre que se
nos den dos polgonos orientados, podemos comparar sus orientaciones; es decir, podemos
decidir si las orientaciones respectivas son iguales (o la misma) o no lo son. As,
Denicion 2.1.0.11. Dos polgonos semejantes son directa o inversamente semejantes si
sus orientaciones correspondientes son iguales o no.
Ejercicios
1. Demuestra que si dos pares de lados correspondientes son proporcionales y los angulos
incluidos son congruentes entonces los tri angulos son semejantes.
2. Demuestra el Corolario 2.1.0.9 (de Thales), as como su recproco.
3. Demuestra el Corolario 2.1.0.10.
2.2. Teorema de Tolomeo
Anteriormente se vio un criterio de ciclicidad referente a los angulos opuestos de un
cuadrilatero. La nocion de antiparalelismo de pares de rectas nos da un segundo criterio a
este respecto.
Denicion 2.2.0.12. Dos rectas son antiparalelas con respecto a otras dos si las primeras
forman angulos internos iguales con la bisectriz de las otras dos.
Lema 2.2.0.13. Si un par de rectas es antiparalelo respecto a otro par, entonces el segundo
par tambien es antiparalelo respecto al primero.
As, nos referiremos a dos pares de rectas antiparalelas cuando alguno de los dos pares
es antiparalelo respecto del otro.
28 Rodolfo San Agustn Chi
Teorema 2.2.0.14. Los puntos en que dos pares de rectas antiparalelas se cortan mutu-
amente son concclicos y, recprocamente, los pares de lados opuestos de un cuadril atero
cclico forman pares de rectas antiparalelas.
Vemos que no es posible hallar un criterio de ciclicidad de puntos referente unicamente
a los lados de un cuadrilatero. Sin embargo, el teorema que sigue nos da un tercer criterio
de ciclicidad de puntos referente a los lados y las diagonales de un cuadrilatero.
Teorema 2.2.0.15. (Ptolomeo) En un cuadril atero cclico la suma de los productos de
pares de lados opuestos es igual al producto de las diagonales.
En smbolos, si ABCD es un cuadrilatero cclico, entonces
AB CD +BC DA = AC BD. (2.2)
De hecho, el recproco del teorema tambien es valido
As, probaremos que un cuadrilatero ABCD es cclico si y solamente si se verica la
formula (2.2).
Demostraci on.
(1) Sea ABCD un cuadrilatero. Hipotesis
(2) Sea l una recta tal que (l, AB) CAD Hipotesis
(3) Sea m una recta tal que (AB, m) ADC Hipotesis
As,
(4) Sea E = l m
(5) EAB CAD corolario 2.1.0.7,d (2, 3, 4)
(6)
EA
CA
=
AB
AD
=
BE
DC
(5)
(7) EAC BAD corolario 2.1.0.7,e (2, 6)
(8)
EA
BA
=
AC
AD
=
CE
DB
(7)
(9) As, ABCD es cclico sii EB +BC = EC (3) y (4)
(10) i.e. sii
AB CD
AD
+BC =
AC BD
AD
(6), (8) y (9)
(11) i.e. sii AB CD +BC AD = AC BD. (10)
El caso de los trapezoides, en particular los trapecios y rect angulos, al tener pares de
lados paralelos, no pareciera estar considerado en este teorema. Sin embargo, recordemos
que la bisectriz de un angulo es la recta equidistante de sus lados. En el caso de rectas
paralelas la recta equidistante juega el papel de la bisectriz para rectas concurrentes. Con
esto podemos redenir antiparalelismo entre pares de rectas de tal forma que tambien se
incluya el caso de paralelas: Dos rectas son antiparalelas con respecto a otras dos si las
primeras forman angulos internos iguales con la recta equidistante de las otras dos.
Ejercicios
1. Demostrar el Lema 2.2.0.13
29 Notas de Geometra Moderna I
2. Si cuatro o m as tri angulos directamente semejantes son tales que un conjunto de
vertices correspondientes coinciden y un segundo conjunto es concclico, el tercer con-
junto tambien es concclico.
3. Supon que A, B, C y D son los vertices de un cuadrilatero inscrito en una circunfer-
encia. Demuestra que si DB es un di ametro, BDA = x y CDB = y, entonces
sen(x +y) = sen xcos y + cos x sen y.
4. Construye un cuadrilatero cclico dados sus cuatro lados.
5. Haciendo uso del Teorema de Ptolomeo encuentra la raz on de la diagonal de un
pentagono regular a su lado. Con ayuda de esta raz on, demuestra que
cos 36

5 + 1
4
.
6. Usando el resultado anterior, inscribe un pentagono regular en una circunferencia
dada y prueba que la raz on de sus lados al radio es
1
2

10 2

5.
7. Construir un pentagono regular dado
a) su lado.
b) su diagonal.
2.3. Homotecia
Sean M una gura que consta de puntos A, B, y an alogamente para M

junto con
una correspondencia biyectiva M M

.
Denicion 2.3.0.16. Se dice que M y M

son (guras) homoteticas si hay un punto O en


el plano y un n umero real k tales que para cada par A y A

de puntos correspondientes,
1. O, A y A

est an alineados
2.
OA

OA
= k
El punto O es el centro de la homotecia y la constante k es la raz on de dicha homotecia.
Denotamos con el smbolo (O, k) a la homotecia con centro O y raz on k vista como una
funcion
plano O plano O
A A

30 Rodolfo San Agustn Chi


lo cual es una funcion biyectiva. As, mediante la extencion natural que sigue:
O O
tenemos una funcion biyectiva denida en todo el plano.
Tambien podemos interpretar a dicha funcion como la dilataci on del plano con centro
O y raz on de dilataci on k.
Considerando cualitativamente el valor de k tenemos que:
1. Si 0 < k < 1 se trata de una contracci on.
2. Si k = 1, se trata de la funcion identidad.
3. Si 1 < k se trata de una expancion.
4. El caso k = 1 corresponde a la simetra del plano con respecto al punto O.
5. As, si k < 0, considerando la composici on de funciones, tenemos que
(O, k) = (O, [k[) (O, 1).
2.3.1. Algunas propiedades basicas de la homotecia:
Propiedad 2.3.1.1. Una homotecia con k ,= 1 mueve a todos los puntos del plano, salvo
al centro de dicha homotecia
Demostraci on.
Si P ,= O,
OP

OP
= k ,= 1 =OP

,= OP
P

,= P.
Por otra parte, si O O

la raz on
OO

OO
no est a denida.
Observaci on 2.3.1.2.
Consideremos dos puntos distintos P y M, digamos.
As,
OP

OP
=
OM

OM
= k por denicion de homotecia. y los tri angulos OPM y OP

son semejantes con raz on de semejanza k.


A partir de esta observacion, las propiedades siguientes se prueban directamente:
Propiedad 2.3.1.3.
1. Una homotecia manda rectas en rectas paralelas.
De hecho, basta vericar que manda segmentos de recta en segmentos de recta
paralelos; as mismo,
2. Una homotecia dilata magnitudes de segmentos de acuerdo a la raz on de dicha ho-
motecia.
31 Notas de Geometra Moderna I
3. Una homotecia manda circunferencias en circunferencias.
De hecho, vemos que la homotecia (O, k) manda a la circunferencia de centro A y
radio r en la circunferencia de centro A

y radio r

:= rk.
4. Una homotecia preserva angulos.
Observaci on 2.3.1.4. De lo anterior tenemos que la homotecia manda tri angulos en
tri angulos semejantes, de donde tri angulos homoteticos son semejantes. Sin embargo, la
armacion recproca no es cierta en general.
Ejemplo 2.3.1.5. Sean N, L y M los puntos medios de los lados del tri angulo ABC,
respectivamente. Entonces, los tri angulos LMN y ABC son homoteticos:
Demostraci on.
(1)
BL
LC
=
CM
MA
=
AN
NB
= 1
(2) MN | BC por el lema 2.1.0.1 en (1)
(3)

MGN BGC,
donde G := BM CN
por el teorema 2.1.0.5 (2)
(4)
GM
GB
=
GN
GC
=
MN
BC
=
1
2
pues

ANM ABC,
con
BC
NM
=
AB
AN
= 2
(5)

la homotecia (G,
1
2
) manda
B M y C N.
(4)
Pregunta: A ?
Respuesta: BA ;c, donde

c | BA
y M c.
es decir, c = LM.
Analogamente, CA ;LN
A (= BA CA) LM LN = L
Observaci on 2.3.1.6. As, tenemos que G, el centro de gravedad de un tri angulo, es un
punto de trisecci on de las medianas.
2.3.2. Circunferencias homoteticas
Armacion 2.3.2.1. Dos circunferencias no concentricas son homoteticas de dos maneras
diferentes.
Demostraci on.
Sean c(O, r) y c(O

, r

) dos circunferencias con O ,= O

,
32 Rodolfo San Agustn Chi
A
B C
M
N
G
L
?
Figura 2.4:
Sean K y H los puntos que dividen al segmento OO

en las razones
r
r

y
r
r

, respec-
tivamente.
As, para las homotecias (K,
r

r
) y (H,
r

r
) tenemos que
KO

KO
=
r

r
(2.3)
HO

HO
=
r

r
(2.4)
Denicion 2.3.2.2. Los centros de dichas homotecias, K y H, respectivamente, son los
centros de similitud, externo e interno, de dichas circunferencias.
Es decir, los centros de similitud de las circunferencias con centros O y O

dividen al
segmento OO

el la raz on de los radios.


Teorema 2.3.2.3. Si dos circunferencias tienen tangentes externas comunes, estas pasan
por uno de sus centros de homotecia y si tienen tangentes internas comunes, estas pasan
por el otro centro de homotecia.
H
K
O O
Figura 2.5: Las tangentes comunes pasan por los centros de similitud.
33 Notas de Geometra Moderna I
2.3.3. Puntos hom ologos y antihom ologos
Sea l una recta que pasa por uno de los centros de similitud de dos circunferencias y
que corta a estas en A, B y A

, B

, respectivamente. Supongamos que A y A

son puntos
homoteticos (ver la gura 2.6).
Denicion 2.3.3.1.
Los pares de puntos A y A

, as como B y B

son pares de puntos hom ologos.


Los pares de puntos A y B

, as como B y A

son pares de puntos antihom ologos.


A continuaci on tenemos algunas propiedades de este tipo de pares de puntos.
Propiedades 2.3.3.2.
1. B

es paralela a BD y los tri angulos KB

y KBD son semejantes.


2. A

es antiparalela a BD con respecto a A

B y C

D y los tri angulos KA

y KDB
son inversamente semejantes. El cuadril atero A

DB es cclico.
3. El producto de los segmentos KA

y KB es constante.
4. Cada uno de los conjuntos de puntos A

, C

, D, B y A, C, D

, B

es concclico.
5. Las tangentes a las circunferencias en A

y B forman angulos iguales con la recta


A

B. Si dichas tangentes se intersectan en E, el tri angulo EA

B es is osceles.
K
C
D
C
D
E
B
A
B
A
Figura 2.6: Pares de puntos hom ologos y antihom ologos.
Solamente vericaremos la ultima armacion:
Demostraci on. En la gura 2.6:
34 Rodolfo San Agustn Chi
(1) ABD KB

(2) EA

KB

por el corolario (1.3.1.2)


(3) ABD EA

(1) y (2)
(4) DBE DAB D

por el corolario (1.3.1.2)


Finalmente,
(5) ABE ABD +DBE
EA

+D

(3) y (4)
EA

.
2.3.4. Circunferencia de similitud
Denicion 2.3.4.1. La circunferencia de similitud de dos circunferencias no concentricas
es la circunferencia que tiene como di ametro el segmento que une sus centros de similitud.
K H O O
Figura 2.7: Circunferencia de similitud de dos circunferencias dadas.
Teorema 2.3.4.2. La circunferencia de similitud de dos circunferencias no concentricas
es el lugar geometrico de los puntos
1. tales que las razones de sus distancias a los centros de las circunferencias son iguales
a la razon entre los radios y
2. desde las cuales las dos circunferencias subtienden angulos iguales.
Demostraci on.
35 Notas de Geometra Moderna I
Sea C la circunferencia de similitud de las circunferencias c(O, r) y c(O

, r

).
Sea P un punto tal que
PO
PO

=
r
r

. (ver g. 2.8)
Como tambien
r
r

=
OH
HO

, entonces PH es la bisectrz interna del OPO

.
Analogamente, PK es la bisectrz externa del mismo angulo.
PH PK y P c.
Recprocamente, sea P un punto de C.
Sea O

OO

tal que PH sea la bisectrz interna del angulo O

PO

.
Asi, PK es la bisectrz externa del mismo angulo.
De donde
O

H
HO

=
O

K
KO

.
Pero tambien tenemos que
OH
HO

=
OK
KO

y, de ambas ecuaciones, se sigue que


HO

K
=
HO
OK
.
O

= O y
PO
PO

=
r
r

.
P
O O
H K
Figura 2.8: C = p :
PO
PO

=
r
r

Para la segunda parte:


Sea P un punto de la circunferencia de similitud.
As, si PT y PT

son tangentes a c(O, r) y c(O

, r

), respectivamente, por la primera


parte,
PO
PO
=
r
r

=
OT
O

y, por el corolario 2.1.0.10, los tri angulos PTO y PT

son semejantes.
En particular, TPO TP

y, de aqu, la armacion.
Si dos circunferencias tienen radios iguales, su circunferencia de similitud es la mediatriz
del segmento OO

, conjuntamente con la recta al innito.


36 Rodolfo San Agustn Chi
T
T
O O
P
Teorema 2.3.4.3. Si dos circunferencias tienen circunferencia de similitud C, cualquier
circunferencia que pase por los centros de las primeras es ortogonal a C.
Demostraci on.
En la gura (2.9):
O K
P
O H
Figura 2.9:
Sea MP la tangente a la circunferencia por P, O y O

con el punto M sobre la recta


OO

. Queremos demostrar que M es el centro de la circunferencia de similitud.


De acuerdo al inciso 2 del corolario (1.3.1.2), MPO MO

P, tenemos que MPO


MO

P y de aqu que
MP
MO

=
OM
PM
. Es decir, tenemos que
MP
2
= MO MO

Como solamente hay una circunferencia ortogonal a otra dada


1. dado el punto de corte P y
2. con centro (M) sobre una recta dada (OO

) (si dicha recta no es la tangente por P)


terminamos
37 Notas de Geometra Moderna I
2.3.5. Crculo de Apolonio
Teorema 2.3.5.1. El lugar geometrico de los puntos cuyas razones de distancias a dos
puntos jos es una constante es una circunferencia.
Demostraci on. Si los puntos jos son O y O

y la raz on de distancias es
r
r

, entonces se
trata de la circunferencia de similitud de c(O, r) y c(O

, r

).
Dicha circunferencia C se conoce como el crculo de Apolonio (asociado a los datos
O, O

y
r
r

).
Observaci on 2.3.5.2. En dicha situaci on, hay una innidad de parejas de circunferencias
c(O, r
i
), c(O

, (r
i
)

) asociadas al crculo de Apolonio C, con


r
i
(r
i
)

=
r
r

.
Ejercicios
1. Si dos circunferencias se intersectan, las rectas del punto de intersecci on a los centros
de similitud bisectan los angulos formados por los radios trazados a ese punto.
2. La bisectriz del angulo en A del tri angulo ABC corta a BC en L.
Si C describe una circunferencia cuyo centro es A y B permanece jo, cual es el lugar
geometrico de los puntos L?
3. La distancia entre los centros de dos circunferencias, cuyos radios son a y b, es c.
Encuentra el centro de la circunferencia de similitud.
4. Dos circunferencias se intersectan en los puntos A y B. Una recta variable por A
intersecta a las circunferencias en P y Q. Si R divide al segmento PQ en una raz on
dada, demuestra que el lugar geometrico de los puntos R es una circunferencia.
5. Encontrar un punto tal que sus distancias a tres puntos dados tengan razones dadas.
Discutir ampliamente el n umero de soluciones.
6. Construir un rect angulo semejante a un rect angulo dado teniendo la suma de un lado
y una diagonal.
7. Dos rectas dadas se intersectan en un punto inaccesible A.
Se requiere: por un punto P trazar la recta PA.
8. Construir un tri angulo que es semejante a un tri angulo dado y cuyos vertices esten
en tres rectas paralelas dadas.
9. Construir un tri angulo teniendo un lado, el angulo opuesto a dicho lado y la raz on de
los otros dos lados.
10. Construye un tri angulo teniendo un angulo, la suma de los dos lados componentes de
dicho angulo y la suma de otro par de lados.
38 Rodolfo San Agustn Chi
Captulo 3
Algunos teoremas de concurrencia
y de colinealidad
Hubo una epoca en que el estudio de las conguraciones estaba
considerado como la rama mas importante de toda la geometra
D. Hilbert
1
-S. Cohn-Vossen.
2
Esta frase parece increible hoy en dia, en que la geometra parece estar en vias de
extincion. Sin embargo, desde un punto de vista algortmico o computacional, las cosas se
ven de forma diferente. Las aplicaciones
El objetivo principal de este captulo es presentar algunos primeros criterios sobre
colinealidad (de puntos) y concurrencia (de rectas) como son los teoremas de Ceva, Menelao,
Pappus y Desargues.
Como ya hemos visto, los vertices y el punto mediano de un tri angulo, junto con los
lados y las medianas de dicho tri angulo, contituyen un conjunto de cuatro puntos y seis
rectas tales que:
1. Cada recta contiene a dos de esos puntos y
2. Por cada uno de los puntos pasan tres de las rectas.
As, tenemos que el teorema que da la existencia del punto mediano de un tri angulo
completa la existencia de esta conguracion
3
de puntos y rectas.
En general, una conguracion de puntos y rectas consta de
1
David Hilbert, 1862-1943.
2
[HCV52], p. 95. Este libro y, en particular, la frase anterior han sido una inspiraci on de muchos
matematicos para estudiar conguraciones. Sin embargo, la frase citada parece ser una gran exageracion
pues, si bien durante el periodo 1840-1930 hubo una buena cantidad de matematicos interesados en el tema,
este no parece haber sido un tema central de investigaci on, a juzgar tanto por la abundancia relativa de las
publicaciones de la epoca como por su inuencia tanto dentro como fuera de la geometra.
3
Theodore Reye (1838-1919) acu no el termino conguraci on (Kongurazion, en su libro Die Geometrie
der Lage, vols. I, II y III, Leipzig, 1909.
39
40 Rodolfo San Agustn Chi
(i) Un n umero nito v de puntos y
(ii) Un n umero nito b de rectas
tales que
(a) Cada una de las recta contiene a un n umero jo k de los puntos y
(b) Por cada uno de los puntos pasan exactamente r de las rectas mencionadas.
3.1. Teoremas de Ceva y Menelao
Teorema 3.1.0.3 (Ceva).
4
En un tri angulo ABC cualquiera, considerense los puntos L,
M y N en las rectas de los lados BC, AC y AB, respectivamente.
Entonces las rectas AL, BM y CN son concurrentes (o paralelas) si y solamente si
AN
NB

BL
LC

CM
MA
= 1. (3.1)
S
A
R
O
N
M
L
C
B
Figura 3.1:
Demostraci on.
4
Giovanni Ceva, 1648-1734.
41 Notas de Geometra Moderna I
(1) Sea l la paralela a la recta BC por el punto A.
(2) Si las rectas mencionadas concurren en un punto O, digamos.
(3.1) Si O no es un punto ordinario tenemos que BM | NC
y de aqu que
AN
NB
=
AC
CM
.
Tambien LA | BM y, nuevamente,
BL
LC
=
MA
AC
.
As,
AN
NB

BL
LC

CM
MA
=
AC
CM

MA
AC

CM
MA
= 1.
(3.2) Si O es un punto ordinario,
sean R :=

BM l y S :=

CN l.
(3.2.1) AMS BMC =
AN
BN
=
SA
CB
.
(3.2.2) AMR CMB =
CM
AM
=
BC
RA
.
(3.2.3)
BOL ROA =
BL
RA
=
OL
OA
COL SOA =
CL
SA
=
OL
OA

=
BL
RA
=
CL
SA
As,
AN
NB

BL
LC

CM
MA
=
SA
BC

AR
SA

BC
AR
= 1.
Recprocamente,
Sea O :=

BM

CN y sea L

:=

AO

BC
Ya que

BM,

CN y

AL

concurren en O, por la primera parte del teorema, tenemos


que
AN
NB

BL

C

CM
MA
= 1.
Por hip otesis,
AN
NB

BL
LC

CM
MA
= 1
y, de ambas ecuaciones, tenemos que
BL
LC
=
BL

C
es decir, L = L

Teorema 3.1.0.4 (Forma trigonometrica del teorema de Ceva).


Con la nomenclatura del teorema anterior,
Las rectas AL, BM y CN son concurrentes (o paralelas) si y solamente si
senACN
senNCB

senBAL
senLAC

senCBM
senMBA
= 1 (3.2)
Demostraci on.
Las rectas AL, BM y CN son concurrentes (o paralelas)
42 Rodolfo San Agustn Chi
si y solamente si
AN
NB

BL
LC

CM
MA
= 1 Por la ec. (3.1)

CAsenACN
BCsenNCB

ABsenBAL
CAsenLAC

BCsenCBM
ABsenMBA
= 1 Por el teo. (1.4.1.3)

senACN
senNCB

senBAL
senLAC

senCBM
senMBA
= 1
Denicion 3.1.0.5. Un cuadrangulo completo consta de
1. Cuatro puntos no alineados por ternas.
2. Las seis rectas que ellos determinan por pares.
Los puntos son los vertices y las rectas son los lados del cuadrangulo completo. Dos
lados son opuestos si no tienen un vertice en com un. Los tres puntos determinados por las
intersecciones de lados opuestos son los puntos diagonales. El tri angulo determinado por
los puntos diagonales es el triangulo diagonal.
En la gura 3.2, PQR es el tri angulo diagonal del cuadrilatero completo ABCD.
R
C
B
A
D
P
Q
Figura 3.2: Un cuadrangulo completo con su tri angulo diagonal.
Intercambiando los terminos recta y punto:
Denicion 3.1.0.6. Un cuadrilatero completo consta de
1. Cuatro rectas no concurrentes por ternas.
2. Los seis puntos que ellas determinan por pares.
Las rectas son los lados y los puntos son los vertices del cuadrilatero completo. Dos
vertices son opuestos si no est an en un mismo lado. Las tres rectas determinadas por los
pares de vertices opuestos son los lados diagonales. El tril atero determinado por los lados
diagonales es el triangulo diagonal.
43 Notas de Geometra Moderna I
Teorema 3.1.0.7 (Menelao).
5
Con la nomenclatura del teorema de Ceva (3.1.0.3):
los puntos L, M y N estan alineados si y solamente si
AN
NB

BL
LC

CM
MA
= 1 (3.3)
Demostraci on.
Supongamos, primeramente, que los puntos L, M y N est an en una recta l.
(i) Si l es la recta al innito, entonces
AN
NB
=
BL
LC
=
CM
MA
= 1
y de aqu la armacion.
(ii) Si l es una recta ordinaria, considerense P, Q y R, los pies de las perpendiculares
a l por A, B y C, respectivamente.
B
C
L
l
A
N
Q
P
R
M
Figura 3.3:
a) Supongamos que l no es paralela a ninguno de los lados del ABC
As,
ANP BNQ =
AN
NB
=
AP
QB
CMR AMP =
CM
MA
=
RC
AP
BLQ CLR =
BL
LC
=
QB
CR

AN
NB

CM
MA

BL
LC
=
AP
QB

RC
AP

QB
CR
= 1
b) Si l es paralela al lado, digamos BC del tri angulo,
5
Menelao de Alejandra, ca. 70-140.
44 Rodolfo San Agustn Chi
Entonces
AN
NB
=
AM
MC
y
BL
LC
= 1.

AN
NB

BL
LC

CM
MA
= 1.
Recprocamente,
a) Si las razones
AN
NB
,
CM
MA
y
BL
LC
son todas distintas de 1.
En este caso, sea L

:= NM BC.
As, L

, M y N est an alineados y, por la primera parte del teorema,


AN
NB

BL

C

CM
MA
= 1.
Entonces
BL
LC
=
BL

C
y L = L

.
b) Si
BL
LC
= 1, digamos.
Entonces
AN
NB

CM
MA
= 1 y
AN
NB
=
AM
MB
;
i.e. NM | BC
L, M y N est an alineados.
Teorema 3.1.0.8 (Forma trigonometrica del teorema de Menelao).
Con la nomenclatura del teorema de Ceva (3.1.0.3):
los puntos L, M y N estan alineados si y solamente si
senACN
senNCB

senBAL
senLAC

senCBM
senMBA
= 1. (3.4)
La demostracion es completamente an aloga a la del teorema (3.1.0.4).
Teorema 3.1.0.9 (De division interna y externa).
Con la nomenclatura del teorema de Ceva (3.1.0.3):
Sea L

:=

MN

BC.
Entonces,
BL
LC
=
BL

C
. (3.5)
i.e. L y L

dividen interna y externamente al segmento BC en la misma raz on


Demostraci on.
Siendo concurrentes las rectas AL, BM y CN, por el teorma de Ceva (3.1.0.3), se tiene
que
AN
NB

BL
LC

CM
MA
= 1.
45 Notas de Geometra Moderna I
B C
N
L L
M
A
Figura 3.4:
BL
LC
=
BL

C
Por otra parte, como L

, M y N est an alineados, ahora, por el teorema de Menelao


(3.1.0.7), se tiene que
AN
NB

BL

C

CM
MA
= 1.
y de ambas ecuaciones tenemos que
BL
LC
=
BL

C
.
Observaci on 3.1.0.10. En el captulo que sigue veremos que este teorema da una con-
struccion proyectiva del conjugado arm onico de un punto con respecto a otros dos.
Una forma de enunciar el teorema de Pappus
6
es la que sigue:
Teorema 3.1.0.11 (Pappus). Si A, C y E son tres puntos alineados y B, D y F tambien,
entonces los puntos L := AB DE, M := CD FA y N := BC EF tambien estan
alineados
Demostraci on.
Consideremos el tri angulo cuyos vertices son los puntos siguientes:
U := EF CD, V := AB EF y W := AB CD.
Aplicando el teorema de Menelao (3.1.0.7) a dicho tri angulo y a las cinco ternas de
puntos alineados siguientes:
LDE, AMF, BCN, ACE y BDF,
tenemos que
V L
LW

WD
DU

UE
EV
= 1,
V A
AW

WM
MU

UF
FV
= 1,
V B
BW

WC
CU

UN
NV
= 1,
6
46 Rodolfo San Agustn Chi
D
V
L
A
E
C
F
B
U
M
W
N
Figura 3.5:
V A
AW

WC
CU

UE
EV
= 1, y
V B
BW

WD
DU

UF
FV
= 1.
Dividiendo el producto de las primeras tres expreciones entre el producto de las dos
restantes tenemos que
V L
LW

WM
MU

UN
NV
= 1;
lo cual es, por el propio teorema (3.1.0.7), el resultado pedido
Otra forma de enunciar dicho teorema es relacion andolo con el teorema que sigue:
Teorema (Pascal). Si un hexagono esta inscrito en una c onica, entonces las intersecciones
de pares de lados opuestos estan en una recta.
Consideremos a los puntos A, B, C, D, E y F como les vertices consecutivos de un
hexagono. La hip otesis del teorema es que dichos vertices esten, alternadamente, sobre dos
rectas. La conclusion del teorema es que las intersecciones de los tres pares de lados opuestos
de dicho hexagono (i.e. los puntos U, V y W) tambien est an alineados. La relaci on con el
teorema de Pascal (3.1) se da al considerar al par de rectas ACE y BDF como una conica
singular, extendiendo as el teorema ya mencionado para dicho tipo de conicas.
Ejercicios
1. Demuestra, utilizando el Teorema de Ceva o el de Menelao que, en cualquier tri angulo,
las alturas son concurrentes.
2. Los seis centros de similitud de tres circunferencias, tomadas por parejas, est an por
ternas en cuatro lneas rectas.
3. Dadas dos rectas paralelas y el segmento AB en una de ellas, encuentra el punto medio
de AB, usando unicamente regla.
47 Notas de Geometra Moderna I
4. Supongamos que las transversales PQR y P

cortan a los lados BC, CA y AB,


respectivamente. Sean X := BC QR

, Y := CA RP

y Z := AB PQ

. Entonces los
puntos X, Y y Z est an alineados.
5. En la gura las rectas SR y BC son paralelas. Demuestra que
OL
AL
+
OM
BM
+
ON
CN
= 1.

'
'
'
'
'
'
'
'
'
'
'
' '
`
`
`
`
`
`
`
`
`
`
`
` `
`
`
`
`
`
`
`
`
`
`
``
q
B
q
L
q
C
q
A
q
M
q O
q
S
q
R
q
N
6. Si P es el punto medio del lado BC del tri agulo ABC y Q y R son puntos cualesquiera
en AC y AB de tal forma que BQ y CR se cortan en AP, entonces QR es paralelo a
BC.
7. Si una circunferencia corta a los lados BC, CA y AB del tri angulo ABC en los puntos
P, P

; Q, Q

; R, R

, respectivamente, y si AP, BQ y CR son concurrentes, entonces


AP

, BQ

y CR

tambien son concurrentes.


8. Sea L, M y N los puntos medios de los lados BC, CA y AB del tri angulo ABC, y
sean D, E y F tres puntos cualesquiera en estos lados para los cuales AD, BE y CF
son concurrentes. Si P, Q, R son los puntos medios de AD, BE, CF respectivamente,
demostrar que PL, QM, RN son concurrentes.
9. Si en el ejercicio anterior, X, Y , Z son los puntos medios de EF, FD, DE respec-
tivamente, demostrar que AX, BY , CZ son concurrentes; y tambien que LX, MY ,
NZ son concurrentes.
3.2. Triangulos en perspectiva
Denicion 3.2.0.12. Dos guras est an en perspectiva si las rectas que unen puntos corre-
spondientes de ambas guras son concurrentes.
Se llama centro de perspectiva al punto donde concurren dichas rectas.
48 Rodolfo San Agustn Chi
Ejemplo 3.2.0.13. Figuras homoteticas est an en perspectiva pero la armacion recproca
no es cierta en general.
Teorema 3.2.0.14 (Desargues). Dos tri angulos estan en perspectiva si y solamente si los
puntos de interseccion de lados correspondientes son colineales.
Se llama eje de perspectiva a la recta de dichos puntos de intersecci on.
A
B
B
C
C
R
Q P
A
O
Figura 3.6:
Demostraci on.
Sean ABC y A

dos tri angulos en perspectiva con el punto O como centro de


perspectiva.
Sean P := AB A

, Q := BC B

y R := CA C

.
La recta B

P es transversal al ABO

AP
PB

BB

O

OA

A
= 1.
Analogamente, la recta B

Q es transversal al CBO

CQ
QB

BB

O

OC

C
= 1.
Finalmente, la recta A

R es transversal al ACO

AR
RC

CC

O

OA

A
= 1.
As, de las tres ecuaciones tenemos que
AP
PB

BQ
QC

CR
RA
= 1.
Recprocamente:
Si los puntos P, Q y R est an alineados entonces los tri angulos AA

R y BB

Q est an en
perspectiva desde P.
49 Notas de Geometra Moderna I
Las intersecciones de lados correspondientes son O := AA

BB

, C y C

.
Por la primera parte del teorema, dichos puntos est an alineados.
i.e., la recta CC

pasa por el punto O


Ejercicios
1. Reere la soluci on del problema siguiente al Teorema de Desargues: Dadas dos rectas
y un punto que no se encuentra en ninguna de las dos, con regla solamente, trazar
una recta a traves del punto dado y del punto de intersecci on de las dos rectas dadas,
sin usar este punto de intersecci on.
2. Si tres tri angulos tienen un centro com un de perspectiva, los tres ejes de perspectiva
son concurrentes.
3. Si las rectas AA

, BB

, CC

son concurrentes, los seis puntos de intersecci on de los


pares de rectas AB, A

; BC, B

; CA, C

; A

B, AB

; B

C, BC

y C

A, CA

se
encuentran por ternas en cuatro lneas rectas.
4. Demuestra que siempre es posible trazar un tri angulo que este en perspectiva con un
tri angulo dado y que sea semejante a otro tri angulo dado.
5. Demuestra que el teorema de Papus implica el teorema de Desargues.
6. Demostrar que dos tri angulos en doble perspectiva est an en triple perspectiva.
7. Demostrar que si los tri angulos ABC y PQR est an en triple perspectiva desde U, V
y W entonces los tri angulos ABC, PQR y UVW forman un sistema de Hesse; i.e. los
vertices de cada uno de ellos son los centros de perspectiva para los dos tri angulos
restantes.
50 Rodolfo San Agustn Chi
Captulo 4
Relaciones arm onicas
4.1. Division arm onica.
Denicion 4.1.0.15. Sean A, B, C y D puntos alineados. El segmento AB est a dividido
(o separado) arm onicamente por los puntos C y D si
AC
CB
=
AD
DB
(4.1)
i.e. si C y D dividen interna y externamente a AB en la misma raz on.
Ejemplo 4.1.0.16.
1. Los centros de similitud de dos circunferencias dividen arm onicamente al segmento de
los centros de dichas circunferencias.
2. Sean l y l

las bisectrices interna y externa del angulo en A de un ABC y sean L y L

los puntos en que estas rectas cortan, respectivamente, al lado BC de dicho tri angulo.
Entonces L y L

dividen arm onicamente al segmento BC.


Demostraci on.
BAL

= BAL

2
L

AC = BAL

2
BAL

AC = 0 mod
sen BAL

= sen L

AC
i.e.
sen BAL

sen L

AC
= 1
Observaci on 4.1.0.17.
51
52 Rodolfo San Agustn Chi
1.
AC
CB
=
AD
DB

CA
AD
=
CB
BD
.
es decir, la separacion arm onica es una relaci on simetrica (recproca) entre las parejas
de puntos A, B y C, D.
En tal caso se dice que A, B, C y D es una hilera arm onica de puntos, que los dos
pares A, B y C, D son pares de puntos conjugados (arm onicos) o bien que dichos
cuatro puntos son arm onicos.
2. Si dos de cuatro puntos arm onicos coinciden un tercero coincide con ellos.
Problema 4.1.0.18. Construir el conjugado arm onico de un punto dado respecto de otros
dos puntos tambien dados.
El algoritmo que sigue resuelve el problema:
Algoritmo 4.1.0.19.
1. Sean l
A
una recta por A y l
B
una recta paralela a la alterior por B.
2. Sea m una recta transversal a las anteriores por C.
3. Sean P := m l
A
y Q := m l
B
.
4. Sea R el punto simetrico de Q respecto de B.
5. Entonces D := PR AB.
m
l
A C B
P
B
R
l
A
D
Q
Figura 4.1:
Demostraci on.
1. APC BQC pues tienen sus lados correspondientes paralelos.
2.
AC
CB
=
AP
QB
.
3. APD BRD pues tienen sus lados AP y BR paralelos y el angulo en D en
com un.
53 Notas de Geometra Moderna I
4.
AD
DB
=
AP
BR
.
5. QB BR por hip otesis.
6.
AC
CB
=
AD
DB
Observaci on 4.1.0.20. Del an alisis del algoritmo anterior se sigue que:
1. Si tres puntos est an alineados el conjugado arm onico de cualquiera de ellos con res-
pecto a los dem as siempre existe y es unico.
2. Si AC = CB, entonces D = .
Propiedades 4.1.0.21.
1. Si una hilera (ordenada) de puntos es armonica, entonces las otras siete hileras (or-
denadas) que se obtienen al permutar los puntos, respetando los pares conjugados,
tambien son armonicas.
2. Si A, B, C y D son armonicos, los segmentos AC, AB y AD estan en progresi on
armonica y recprocamente.
3. Si O es el punto medio del segmento AB y los puntos A, B, C y D son armonicos,
entonces
OB
2
= OC OD
y recprocamente.
Demostraci on.
1. Esto se verica, en cada caso, directamente de una manipulacion adecuada de la
ecuaci on (4.1).
2.
AC
CB
=
AD
DB

CB
(AB)AC
=
BD
AD(AB)

AB AC
AB AC
=
AD AB
AB AD

1
AC

1
AB
=
1
AB

1
AD

1
AC
+
1
AD
=
2
AB
En esta relaci on de segmentos AB es la media arm onica de AC y AB.
54 Rodolfo San Agustn Chi
3.
AC
CB
=
AD
DB

AO +OC
CO +OB
=
AO +OD
OD +BO

OB +OC
CO +OB
=
OB +OD
OD +BO
(OB +OC)(OD +BO) = (OB +OD)(CO +OB)
2 OC OD = 2 OB
2
OB
2
= OD OC
Observamos que, si A, B; C, D es una hilera arm onica de puntos y O es cualquier punto
fuera de la recta por A, B, C y D, entonces, por el teorema generalizado de la bisectrz,
tenemos que
senAOC
senCOB
=
senAOD
senDOB
.
Con esto, se ocurre denir
Denicion 4.1.0.22. Un haz de rectas OA, OB, OC y OD es arm onico si
senAOC
senCOB
=
senAOD
senDOB
.
Observaci on 4.1.0.23. Tal como en el caso de hileras de puntos, tenemos que
1. La separacion arm onica de haces de rectas es una relacion simetrica (recproca).
Esto por la observacion 4.1.0.17.
2. Si tres rectas son concurrentes la conjugada arm onica de cualquiera de ellas con res-
pecto a las dem as siempre existe y es unica.
Esto por el problema 4.1.0.18.
3. Si OC es bisectrz del AOB entonces OD es la otra bisectrz del mismo angulo.
Esto por la observacion 4.1.0.20.
4.2. Transversal de un haz arm onico
Denicion 4.2.0.24. Una transversal de un haz es cualquier recta que no pase por el
vertice del haz.
Teorema 4.2.0.25. Las rectas de un haz armonico cortan a cualquier transversal en una
hilera armonica e inversamente.
55 Notas de Geometra Moderna I
Demostraci on. En la gura (4.2),
senAOC
senCOB
=
senAOD
senDOB

OA
BO

senAOC
senCOB
=
OA
BO

senAOD
DOB

AC
CB
=
AD
DB
C B
D A
O
Figura 4.2:
Corolario 4.2.0.26. Si una transversal corta a las rectas de un haz en una hilera armonica,
entonces cualquier otra transversal tambien corta a las rectas de dicho haz en una hilera
armonica.
4.3. Hileras arm onicas en perspectiva
Teorema 4.3.0.27. Si las hileras armonicas A, B, C, D y A, B

, C

, D

estan en rectas
distintas entonces:
1. las rectas BB

, CC

y DD

son concurrentes y
2. las rectas BB

, C

D y CD

tambien son concurrentes.


Demostraci on.
1.- Sean O := BB

CC

y D

:= OD AB

.
Entonces, por el corolario 4.2.0.26, la hilera A, B

, C

, D

es arm onica y, por la unicidad


del cuarto arm onico (Obs. 4.1.0.20), D

= D

.
2.- Ya que la hilera A, B

, C

, D

es arm onica, tambien A, B

, D

, C

lo es, por la
propiedad 4.1.0.21.
De aqu, procedemos como en el caso anterior.
56 Rodolfo San Agustn Chi
A
B C D
O
C
B
D
O
Figura 4.3:
4.4. Rectas conjugadas perpendiculares
Teorema 4.4.0.28. Si en un haz armonico de rectas diferentes un par de rectas es perpendi-
cular, entonces estas rectas bisecan los angulos formados por las otras dos y recprocamente.
Demostraci on. De acuerdo a la gura (4.4),
O
A
A
C
C B
B
D
l
A
Figura 4.4:
57 Notas de Geometra Moderna I
(1) Sea O(ABCD) un haz arm onico con OC OD. Hipotesis.
(2) Sea l | OD tal que O / l. Hipotesis.
(3) Sean

= l OA,
B

= l OB y
C

= l OC.
Hipotesis.
(4) La hilera A

, B

; C

, es arm onica Por (1).


(5) A

Por (4).
(6) OC

l Por (1) y (2)


(7) AOC BOC Ax. III.5 en (5) y (6).
(8) BOD A

OD De (7).
Recprocamente:
(1) Sean OC y OD las bisectrices del AOB.
(2) senAOC = senCOB AOC = COD
(3) senAOD = sen DOA

AOD +DOA

=
(4) = sen BOD DOA

= BOD
(5) O(ABCD) es arm onico De (2), (3) y (4)
4.5. Curvas ortogonales
Denicion 4.5.0.29. El angulo de intersecci on de dos curvas en un punto es el angulo
formado por las tangentes respectivas en dicho punto
Propiedades 4.5.0.30.
1. Si dos circunferencias se intersecan los angulos de intersecci on en sus puntos comunes
son iguales.
2. Si dos circunferencias son ortogonales, una tangente a una de ellas en un punto de
interseccion pasa por el centro de la otra. Si el radio de una de las circunferencias
trazado a un punto com un es tangente a la otra, las circunferencias son ortogonales.
3. El cuadrado de la distancia entre los centros de dos circunferencias ortogonales es
igual a la suma de los cuadrados de sus radios.
La primera de estas propiedades se puede ver considerando que la gura es simetrica
respecto a la linea de los centros. Las dos restantes se apoyan en el hecho de que la tangente
a una circunferencia es ortogonal al radio trazado al punto de tangencia.
Teorema 4.5.0.31. Una circunferencia con diametro AB es ortogonal a cualquier circun-
ferencia que pase por C y D, un par de conjugados armonicos de A y B y recprocamente.
58 Rodolfo San Agustn Chi
Figura 4.5:
O
C
B
D
A
P
Figura 4.6:
59 Notas de Geometra Moderna I
Demostraci on. En la gura 4.6, AB es un di ametro y OP es el radio de una de las circun-
ferencias trazado a P, uno de los puntos de intersecci on de ambas circunferencias.
Ya que OB
2
= OC OD, por la propiedad (4.1.0.21, (3)), tenemos que OP
2
=
OC OD y de aqu que, con la observacion (2.1.0.8), el radio OP sea tangente a la segunda
circunferencia.
Ejercicios
1. La bisectriz del angulo en A del tri angulo ABC corta al lado opuesto en P. Q y R
son los pies de las perpendiculares desde B y C sobre AP. Demuestra que los cuatro
puntos A, P, Q, R son arm onicos.
2. O es un punto cualquiera de la altura AD del tri angulo ABC. BO y CO intersctan a
AC y AB en E y F respectivamente. Prueba que el angulo EDF est a bisectado por
DA.
3. A, B, C, D son cuatro puntos en una lnea recta. Encuentra dos puntos que sean
conjugados arm onicos con respecto a A y B as como con respecto a C y D. Elabora
una discusion completa de los casos.
4. P y Q son los centros de dos circunferencias que tienen tangentes exteriores comunes
que se intersectan en R y tangentes interiores comunes que se intersctan en S. Demues-
tra que existen circunferencias con centros R y S cuyas tangentes comunes exteriores
se intersectan en P y cuyas tangentes comunes interiores se intersectan en Q.
5. Las tangentes a una circunferencia en P y Q se intersectan en A y la recta de di ametro
BC pasa por A. Demuestra que A y Q est an separados arm onicamente por los puntos
en los cuales su recta es intersectada por PB y PC.
4.6. Cuadrangulos y cuadrilateros completos
Recordemos que un cuadrangulo completo consta de

Cuatro puntos no alineados por ternas.


Las seis rectas que ellos determinan por pares.
Los puntos son los vertices y las rectas son los lados del cuadrangulo completo.
Dos lados son extbfopuestos si no tienen un vertice en com un.
Los tres puntos determinados por las intersecciones de lados opuestos son los puntos
diagonales.
El tri angulo determinado por los puntos diagonales es el triangulo diagonal.
Analogamente, intercambiando los terminos recta y punto:
Un cuadrilatero completo consta de

Cuatro rectas no concurrentes por ternas.


Los seis puntos que ellas determinan por pares.
60 Rodolfo San Agustn Chi
Las rectas son los lados y los puntos son los vertices del cuadrilatero completo.
Dos vertices son opuestos si no est an en un mismo lado.
Las tres rectas determinadas por los pares de vertices opuestos son los lados diago-
nales.
El tril atero determinado por los lados diagonales es el triangulo diagonal.
4.6.1.
Teorema 4.6.1.1. En cada diagonal de un cuadril atero completo: La hilera determinada
por los dos vertices en ella y los puntos en los cuales la intersectan las otras dos es armonica.
Demostraci on. En la gura 4.7 :
C
F
P
A
Q
E
B
D
R
Figura 4.7:
Construyendo el conjugado arm onico del punto Q respecto de B y D: En el tri angulo
ABD, ya que las rectas AQ, BE y DF concurren en el punto C y, tambien las rectas FE
y BD concurren en R, los puntos Q y R son conjugados arm onicos respecto a B y D.
De manera dual:
Teorema 4.6.1.2. En cada punto diagonal de un cuadr angulo completo: El haz determinado
por los dos lados que determinan a dicho punto y las rectas con las cuales se une a los otros
dos puntos diagonales es armonico.
Problema 4.6.1.3. Construir un cuadrangulo completo que tenga el mismo tri angulo dia-
gonal que un cuadrilatero completo dado.
Demostraci on.
En la gura 4.8:
Tenemos el cuadrilatero completo cuyos lados son ABC, AEF, BDE y CDF.
Consideremos alguno de los vertices, digamos X, y su lado opuesto, Y Z en este caso,
del tri angulo diagonal de dicho cuadrilatero.
De acuerdo al teorema 4.6.1.1, sobre dicho lado tenemos la hilera arm onica
C, E; Z, Y (4.2)
61 Notas de Geometra Moderna I
S
A
C
R
F
P
Y
E
E
Z
C
B
X
C
E
D
Q
Figura 4.8:
62 Rodolfo San Agustn Chi
1. La hilera C

, E; A, F est a en perspectiva desde X con la anterior.


Dicha hilera est a en uno de los lados del cuadrilatero dado.
De acuerdo al corolario 4.2.0.26, las rectas CC

, ZF y Y A concurren en un cierto
punto P.
2. Analogamente, las hileras (estando una de ellas en cada lado del cuadrilatero dado)
C

, E; B, D, C, E

; B, A y C

, E

; F, D est an en perspectiva con la hilera


(4.2) y, por lo tanto, las ternas de rectas CC

, ZB, Y D; EE

, ZF, Y D y EE

, ZB,
Y A concurren en puntos Q, R y S, respectivamente.
As, el cuadrangulo completo con vertices P, Q, R y S resuelve el problema.
Ejercicios
1. Construye un cuadrilatero completo que tenga un tri angulo diagonal dado. Se puede
dibujar m as de uno de estos cuadrilateros?
2. Pueden ser colineales los puntos diagonales de un cuadrangulo completo?
3. Demuestra que los puntos medios de las diagonales de un cuadrilatero completo son
colineales.
Sugerencia. En la Figura (3): P, R y Q son los puntos medios de las diagonales
AB, CD y EF, respectivamente. Sean L, M y N los puntos medios de los lados del
tri angulo CEB. Demuestra que M, N y P son colineales, as como M, L, Q y N, L,
R.

/
/
/
/
/
/
/
/
/
/
/
//
`
`
`
`
`
`
`
`
`
`
`
``

q
A
q
B
q C
q
D
qE
q
F
q P
q Q
q
R
q
L
qM
q
N
Figura 4.9:
Demuestra que
MP
PN
=
EA
AC
, etcetera, y que A, F y D son puntos colineales del
tri angulo ECB. Usa el Teorema de Menelao.
63 Notas de Geometra Moderna I
4. Si dos cuadrangulos completos est an en tal forma que los puntos de intersecci on de
cinco pares de lados correspondientes est an en una recta, entonces el punto en el cual
el sexto par de lados se intersecta, tambien est a en esa recta y las cuatro rectas que
unen vertices correspondientes son concurrentes.
5. Cada uno de los tri angulos cuyos lados son tres de las cuatro rectas de un cuadrilatero
completo est a en perspectiva con el tri angulo diagonal del cuadrilatero.
6. Los vertices de un cuadrangulo completo son los tres vertices de un tri angulo y el
punto de intersecci on de sus medianas. Construye su tri angulo diagonal. Tambien
construye un cuadrilatero completo que tenga el mismo triangulo diagonal.
64 Rodolfo San Agustn Chi
Captulo 5
La circunferencia de los nueve
puntos y la recta de Simson
5.1. Grupos ortocentricos
En esta secci on agruparemos varios hechos relacionados con el incentro y los excentros
de un tri angulo dado.
Notaci on 5.1.0.4.
1. A, B, C son los vertices del tri angulo.
2. a, b y c son las longitudes de los lados. BC, CA, y AB, respectivamente.
3. D, E, F son los pies de las alturas.
4. L, M, N son los puntos medios de los lados BC, CA, y AB, respectivamente.
Recordemos que, en un tri angulo cualquiera:
1. El circuncentro O es el punto de intersecci on de las mediatrices de los lados del
tri angulo. Es el centro de la circunferencia circunscrita o circuncrculo.
2. El incentro I es el punto de intersecci on de las bisectrices de los angulo interiores del
tri angulo. Es el centro de la circunferencia inscrita o incrculo. r denota al radio de
dicha circunferencia.
3. Los excentros I
1
, I
2
e I
3
son los puntos de intersecci on de la bisectriz de un angulo
interior y las bisectrices de los angulos exteriores de los otros dos vertices. Son los
centros de las circunferencias circunscritas o excrculos. r
1
, r
2
y r
3
son los radios de
dichas circunferencias, respectivamente.
4. El ortocentro H es el punto de intersecci on de las alturas.
El tri angulo pedal. Sus vertices son los pies de las alturas.
65
66 Rodolfo San Agustn Chi
5. El centro de gravedad, centroide o punto mediano G es el punto de intersecci on de las
medianas.
Teorema 5.1.0.5.
1. Cada lado de un tri angulo es, con respecto a los otros dos lados, antiparalela a aquel
lado del tri angulo pedal cuyos extremos estan en estos dos lados.
2. Las alturas de un tri angulo son las bisectrices de los angulos interiores del tri angulo
pedal.
A
F
E
D
B C
Figura 5.1:
Demostraci on. En la gura 5.1,
La circunferencia de di ametro AB pasa por D y E. As, AB y DE son antiparalelas
con respecto a AC y BC.
Tambien
1. Por lo tanto EDA EBA.
2. Analogamente, ADF ACF.
3. EBA ACF. pues AEB AFC.
4. Por lo tanto, DA es la bisectriz del EDF.
Teorema 5.1.0.6. El area de un tri angulo es el producto del semipermetro y el radio del
crculo inscrito; tambien es el producto del semipermetro disminuido en un lado y el radio
del excrculo correspondiente.
Demostraci on. Si r
1
es el radio del excrculo correspondiente al lado BC, tenemos que
67 Notas de Geometra Moderna I

(IBC) =
1
2
a r
(ICA) =
1
2
b r
(IAB) =
1
2
c r
As,
(ABC) =
1
2
r (a +b +c) = r s.
Analogamente,
(ABC) =
1
2
r
1
(b +c a) = r
1
(s a).
Corolario 5.1.0.7.
1
r
=
1
r
1
+
1
r
2
+
1
r
3
El cuadrangulo completo determinado por el incentro y los excentros de un tri angulo
dado es tal que
Propiedad 5.1.0.8. Cada uno de sus vertices es el ortocentro del tri angulo formado por
los otros tres.
Por esto se le llama el cuadr angulo ortocentrico del tri angulo.
3
I
I
2
A
B
C
I
I
1
Figura 5.2:
En general,
68 Rodolfo San Agustn Chi
Denicion 5.1.0.9. Un conjunto de puntos con la propiedad 5.1.0.8 es un grupo ortocentri-
co de puntos y los cuatro tri angulos que determinan tomando tres puntos a la vez es un
grupo ortocentrico de tri angulos.
Observaci on 5.1.0.10. El tri angulo dado es el tri angulo pedal de cada uno de los cuatro
tri angulos de su grupo ortocentrico de tri angulos; es decir,
Los tri angulos de un grupo ortocentrico tienen el mismo tri angulo pedal.
Teorema 5.1.0.11. El punto medio de un lado de un tri angulo es tambien el punto medio
del segmento determinado por los puntos de contacto de dicho lado con la circunferencia
inscrita y la correspondiente excrita.
N
M
Y
I
Z
B
X
L
X
1
C
I
1
A
Y
1
Z
1
Figura 5.3:
Demostraci on. Sean X, Y y Z los pies de las perpendiculares desde I a los lados del tri angulo
ABC y sean X
1
, Y
1
y Z
1
los pies de las perpendiculares desde I
1
a los lados de dicho
tri angulo.
Probaremos que BX
1
XC (ver la Figura 5.3). De hecho, veremos que cada una de
ambas cantidades es igual a s c.
Primeramente, ya que AZ
1
Y
1
A, BZ
1
BX
1
y X
1
C Y
1
C,
tenemos que AB +BX
1
= X
1
C +CA = s
y de aqu que BX
1
= s c.
Por otra parte; n otese que que AB +XC = s. Entonces tambien tenemos que
XC = s c
69 Notas de Geometra Moderna I
5.2. La circunferencia de los nueve puntos
Teorema 5.2.0.12. Los puntos medios de los lados, los pies de las alturas y los puntos
medios de los segmentos que unen los vertices al ortocentro de un tri angulo cualquiera estan
en una circunferencia.

Esta es la circunferencia de los nueve puntos del tri angulo


1
.
Demostraci on.
En la gura 5.4, P, Q y R son los puntos medios de los segmentos que unen los vertices
al ortocentro H del tri angulo.
B
Q
D L
R
C
N
F
H
A
P
E
M
O
Figura 5.4:
(1) ND es radio de la circunferencia de di ametro AB ADB =

2
(2) DLMN es un trapecio

ND LM
1
2
AB y
NM | BC.
(3) La circunferencia por L, M y N pasa por D (2)
(4) PNDL tambien es inscriptible PNL PDL =

2
(5) La circunferencia por L, M y N pasa por P (4)
(6) Analogamente, dicha circunferencia pasa por
E, F, Q y R.
1
Tambien se le conoce como crculo de Feuerbach, crculo de Euler, crculo de los seis puntos o crculo
medioinscrito.
70 Rodolfo San Agustn Chi
5.3. Recta de Euler
Consideremos primeramente las propiedades de la circunferencia de los nueve puntos
que siguen:
Sea J el centro de dicha circunferencia.
Propiedades 5.3.0.13.
1. El radio de la circunferencia de los nueve puntos es la mitad del radio del circuncrculo
del tri angulo.
2. J es el punto medio entre el ortocentro y el circuncentro. Es decir, H es el otro centro
de homotecia de la circunferencia de los nueve puntos y el circuncrculo.
3. El centro de gravedad G es el otro centro de hometecia de dichas circumferencias.
B
Q
D
L
J
G H
F
N
E
P
A
C
R
O
M
Figura 5.5:
Demostraci on.
1. De acuerdo a la observacion (2.3.1.6), el tri angulo ABC es homotetico al tri angulo
LMN desde G y la raz on de homotecia es
1
2
. Sea g dicha homotecia.
2. De acuerdo al punto anterior tenemos que g(H) = O y g(O) = J.
Como
a) HJ = HG+GJ = 2GO
1
2
=
3
2
GO y
71 Notas de Geometra Moderna I
b) JO = JG+GO =
1
2
GO +GO =
3
2
GO
entonces
HJ
JO
= 1.
2
3. Ya que ANLM es un paralelogramo, la recta AL pasa por el punto medio de MN;
i.e., los tri angulos ABC y LMN tienen el mismo centro de gravedad. Ya que las
homotecias preservan centros de gravedad, dicho punto es el centro de la misma y
de aqu el resultado.
As, la hilera H, G; J, O es arm onica y
Denicion 5.3.0.14. La recta donde se encuentran es la recta de Euler del tri angulo
dado.
Observaci on 5.3.0.15. Los cuatro tri angulos de un grupo ortocentrico tienen la misma
circunferencia de los nueve puntos pues tienen el mismo triangulo pedal.
Por la propiedad (5.3.0.13 1.), los circumradios de estos cuatro tri angulos son iguales.
Sean O
A
, O
B
y O
C
los circumcentros de los tri angulos HBC, AHC y ABH, respecti-
vamente. Consideremos j, la homotecia con centro en J y raz on 1. Se trata de la rotaci on
de radianes en torno al punto J. Dicha transformacion intercambia la altura por el vertice
A con la mediatrz del lado BC del tri angulo (pues intercambia P con L). Por lo tanto,
tambien intercambia los puntos A con O
A
, B con O
B
y C con O
C
.
As, en la gura (5.6), tenemos que los dos grupos ortocentricos siguientes: A, B, C, H
y O, O
A
, O
B
, O
C
, tienen la misma circunferencias de los 9 puntos.
Teorema 5.3.0.16 (Feuerbach).
3
La circunferencia de los nueve puntos es tangente al incrculo y a cada uno de los
excrculos.
2
El argumento alternativo siguiente es muy interesante:
a) PH OL:
1) P es el ortocentro del ANM.
2) O es el ortocentro del LMN.
3) ANM LMN
4) AP OL
5) PH OL, pues AP PH.
b) PH OL
c) PHLO es un paralelogramo.
d) Sea X la intersecci on de sus diagonales;
i.e. X es el punto medio de HO y de PL.
e) X es J, el centro de la circunferencia de los nueve puntos,
pues PDL es un tri angulo rectangulo inscrito en ella.
3
Karl Feuerbach, 1800-1834. Karl Feuerbach, hermano del famoso losofo Ludwig Feuerbach, naci o en
Jena, Alemania, el 30 de mayo de 1800, en una familia protestante. Hijo del jurista Paul Feuerbach y de Eva
Troster, el brillante estudiante de la Universidad de Erlangen, y despues de Friburgo, obtuvo su doctorado
72 Rodolfo San Agustn Chi
B
J
O
C
E
O
B O
M
P
N
F
H
Q
D L
R
C
A
A
O
Figura 5.6: Los grupos ortocentricos A, B, C, H y O
A
, O
B
, O
C
, O.
5.4. La recta de Simson
Teorema 5.4.0.17. Los pies de las perpendiculares a los lados de un tri angulo estan alin-
eados si y solamente si el punto esta en el circumcrculo del tri angulo.
Demostraci on.
En la gura (5.7) tenemos que
(1) CY X CPX CPY X es cclico.
= CPA+APX
y que
(2) AY Z APZ AY PZ es cclico.
= APX +XPZ
Tambien
(3) XPZ +ZBX = BXPZ es cclico.
a los 22 a nos y comenz o a ense nar matematicas en el Gymnasium de Erlangen, frecuentando un crculo de
estudiantes de esta ciudad, conocido por su derroche y sus deudas. Ese mismo a no publico en N uremberg un
librito de 62 pginas de ttulo largo y difuso en el que presenta, en la pagina 38 y demuestra analticamente
el mas bello teorema de geometra elemental descubierto desde la epoca de Euclides, seg un el historiador
J. L. Coolidge :
En un tri angulo,el crculo inscrito es tangente al de Euler.
En 1824, Feuerbach fue detenido y encarcelado durante un a no, junto con 19 estudiantes, en M unchen,
por sus posiciones polticas. Considerandose responsable de todo, sufre una depresion e intenta suicidarse
en dos ocasiones, para salvar a sus compa neros, primero cort andose las venas y despues tir andose por una
ventana. Tras su liberaci on, vuelve a vivir con su familia y gracias a una intervenci on del rey consigue de
nuevo un puesto de profesor en Hof, el cual debera abandonar como consecuencia de una nueva depresion.
En 1828, mejorada su salud, ense na de nuevo en Erlangen, hasta el da en que, desenvainando una espada
en clase, amenaza con cortar la cabeza de los alumnos que no sepan resolver la ecuacion que ha escrito en el
encerado. Abandonando la ense nanza, vivo recluido los seis ultimos a nos de su corta vida dejandose crecer
el pelo, la barba y las u nas, contemplando las pinturas de su sobrino, el pintor Anselmo Feuerbach. Este
profesor impulsivo y perturbado murio en Erlangen, el 12 de marzo de 1834.
73 Notas de Geometra Moderna I
A
B
C
P
Z
Y
X
Figura 5.7:
As, tenemos que
X, Y y Z est an alineados CY X AY Z
CPA XPZ Por (1) y (2)
CPA+ZBX = Por (3)
PABC es cclico
Denicion 5.4.0.18. La recta P(ABC) := XY Z es la recta de Simson del punto P
respecto del tri angulo ABC.
Corolario 5.4.0.19.
1. Si se toman tres cuerdas dibujadas a traves de un punto de una circunferencia como
diametros de circunferencias, estas se intersectan por pares en tres puntos alineados.
2. Recprocamente, si las tres circunferencias que tienen como diametros a los segmentos
PA, PB y PC se intersectan por pares en tres puntos alineados, los puntos P, A, B
y C son concclicos.
Teorema 5.4.0.20. Los circumcrculos de los cuatro tri angulo cuyos lados son los lados
de un cuadril atero completo, tomados tres a un tiempo, tienen un punto en com un.
Demostraci on. Sean a, b, c y d los lados del cuadrilatero y sea c(xyz) el circumcrculo del
tri angulo cuyos lados est an en las rectas x, y y z, respectivamente.
74 Rodolfo San Agustn Chi
(1) Sea a b + M = c(abc) c(abd)
Armacion. El punto M resuelve el problema.
(2) Sean A, B, C y D los pies de las
perpendiculares por M a los lados
a, b, c y d, respectivamente.
(3) A, B y C est an alineados Teorema (5.4.0.17).
(4) Tambien A, B y D est an alineados Teorema (5.4.0.17).
(5) i.e. A, B, C y D est an alineados
(6) M c(cda) (5): C, D y A est an alineados.
(7) M c(cdb) (5): C, D y B est an alineados.
Denicion 5.4.0.21. M es el punto de Miquel
4
del cuadrilatero abcd.
Lema 5.4.0.22. (P(ABC), P

(ABC)) =
1
2
arcPP

Demostraci on. En la gura (5.8),


(1) P(ABC) | AQ Y XP = Y CP = AQP
(2) P

(ABC) | AQ

Analogamente.
(3) (P(ABC), P

(ABC)) =
1
2
arcQQ

(4) =
1
2
arcPP

arcPP

= arcQQ

A
P
P
C
Y
Q
Q
B
X
Figura 5.8:
Corolario 5.4.0.23. Las perpendiculares por Q y Q

a P(ABC) y P

(ABC), respectiva-
mente, se intersectan en el circumcrculo del tri angulo ABC.
Demostraci on.
En general, sean l P(ABC) y l

(ABC).
4
Auguste Miquel
75 Notas de Geometra Moderna I
As, el angulo entre l y l

es el suplemento del angulo entre P(ABC) y P

(ABC).
Es decir, (l, l

) = QAQ

.
Sea T := l l

. As, si Q l y Q

, el cuadrilatero AQTQ

es cclico.
Ya que tanto Q como Q

est an en el circumcrculo del ABC, T tambien lo est a y


terminamos.
Lema 5.4.0.24. La recta de Simson de un punto P biseca al segmento HP.
Demostraci on.
La altura por A corta al circumcrculo en K.
Sean R y T las intersecciones de P(ABC) con PK y PH, respectivamente.
Finalmente, sea S := BC PK
Demostraremos que:
1. R es el punto medio del segmento PS y
2. P(ABC) | HS
y, de aqu, la armacion.
Para probar el inciso 1., en la gura (5.9), queremos probar que RP XR SR.
A
P
B D
S
K
R
X
Z
C
H
T
Figura 5.9:
(1) RXP = ZXP
(2) ZBP PXBZ es cclico.
(3) = ABP
(4) AKP ABKP es cclico.
(5) KPX AK | PX
(6) = RPX
y, por el teorema Pons Asinorum (1.1.0.5), RP XR.
Ahora bien, por (1) y (4) tenemos que AKP ZXP y, como DX PX, con el
mismo teorema (1.1.0.5), RS RX.
Para el inciso (2),
76 Rodolfo San Agustn Chi
(1) BHD BCA BHD BCE
(2) BKD BCA ABKC es cclico.
(3) BHD BKD (1) y (2)
(4) BHD BKD ALA (3,...)
(5) HD DK (4)
(6) HDS KDS LAL (5,...)
(7) HSD KSD (6)
(8) RSX
(9) RXS
Observaci on 5.4.0.25. El punto T est a en la circunferencia de los 9 puntos del tri anguloABC.
Utilisando los lemas 5.4.0.22, 5.4.0.24 y la observacion 5.4.0.25 tenemos
Teorema 5.4.0.26.
1. La recta de Simson de un punto P y la circunferencia de los 9 puntos se cortan en el
punto medio del segmento HP.
2. Las rectas de Simson de dos puntos diametralmente opuestos se cortan ortogonalmente
sobre la circunferencia de los 9 puntos.
Ejercicios
1. En un tri angulo cualquiera, el producto de los dos segmentos en que el ortocentro
divide a la altura es el mismo para las tres alturas.
2. Las seis circunferencias cuyos di ametros son los segmentos que unen por pares los
puntos de un grupo ortocentrico de puntos pasan de cuatro en cuatro por tres puntos.
3. Identifquese el tri angulo diagonal de un cuadrangulo ortocentrico.
4. Las rectas que van de los vertices de un tri angulo a los puntos de contacto de las
circunferencias excritas con los lados opuestos son concurrentes. El punto de concur-
rencia es el punto de Nagel del tri angulo.
5. Con la nomenclatura del texto, demuestrese que los segmentos PO y AL se bisecan.
6. En la gura 5.6, demuestre que los puntos O
B
, P y O
C
est an alineados.
7. Demuestre que los centroides de un grupo ortocentrico de tri angulos forman un grupo
ortocentrico.
8. El circumcrculo biseca a los seis segmentos de recta que unen por pares el incentro y
los tres excentros
9. Construir un tri angulo dados los pies de sus alturas.
77 Notas de Geometra Moderna I
10. Si dos tri angulos est an inscritos en una misma circunferencia, las rectas de Simson se
intersectan a un angulo constante.
11. Construir un tri angulo dados su circumcrculo, el ortocentro y uno de los vertices.
12. Las rectas de Simson de tres puntos con respecto a un tri angulo dado forman un
tri angulo semejante al tri angulo formado por los tres puntos.
13. El cuadrilatero formado por las rectas de Simson de cada vertice de un cuadrilatero
cclico dado respecto del tri angulo formado por los otros tres vertices es semejante al
cuadrilatero dado.
Cierto o falso? Por que?
78 Rodolfo San Agustn Chi
Bibliografa
[AC07] N. Altshiller Court, College Geometry: An introduction to the modern geom-
etry of the triangle and the circle, Dover Publishing Inc., 2007.
[AS80] B.I. Argunov and L.A. Skorniakov, Teoremas de conguraci on., Lecciones
populares de matem aticas., Editorial MIR, Mosc u., 1980.
[BMGO02a] R. Bulajich Manfrino and J.A. G omez Ortega, Geometra, Cuadernos de
olimpiadas de matem aticas, Instituto de matem aticas, 2002.
[BMGO02b] , Geometra. ejercicios y problemas, Cuadernos de olimpiadas de
matem aticas, Instituto de matem aticas, 2002.
[Cro97] P. R. Cromwell, Polyhedra, Cambridge University Press, 1997.
[Euc92] Euclides, Elementos de geometra, Bibliotheca Scriptorum Graecorum et Ro-
manorum Mexicana, vol. I and II, U.N.A.M., 1992.
[Eve69] H. Eves, Estudio de las geometrias, vol. 1, UTEHA, 1969.
[HCV52] D. Hilbert and S. Cohn-Vossen, Geometry and the imagination, Chelsea Pub-
lishing Co., 1952.
[Hea] T.L. Heath, The thirteen books of Euclids Elements, Dover Publications.
[JR07] A. Johnson Roger, Advanced Euclidean Geometry, Dover Publishing Inc.,
2007.
[LMdlESA77] G. Lucio, N. Martnez de la Escalera, and R. San Agustn, Algo de geometra,
Notas de Clase, vol. 155, Facultad de Ciencias, UNAM., 1977.
[Shi61] Levy S. Shively, Introduccion a la geometra moderna, C.E.C.S.A., 1961.
79

Potrebbero piacerti anche